Google Groups no longer supports new Usenet posts or subscriptions. Historical content remains viewable.
Dismiss

Photon Wave-Particle Duality

9 views
Skip to first unread message

Alan

unread,
May 24, 2001, 9:44:03 AM5/24/01
to
Is there a theory states a photon is a particle when interacting with an
electron but a wave when not? If there is not, why?

Thanks,
Alan

Spaceman

unread,
May 24, 2001, 9:53:26 AM5/24/01
to
Waves are made of particles all the time..
stop thinking they are made of "magic" and you will start to learn the
truth.

photons also need electrons to produce this wave.
the medium is known and just ignored badly.

The wave of particles travel over electron highways.
that is all.


"Alan" <kd7...@mac.com> wrote in message news:3B0D0FB7...@mac.com...

Uncle Al

unread,
May 24, 2001, 11:30:11 AM5/24/01
to
Alan wrote:
>
> Is there a theory states a photon is a particle when interacting with an
> electron but a wave when not? If there is not, why?

Particle and wave are limiting cases of observed reality. They are no
more meaningful as heuristics than thinking benzene alternates between
two cyclohexatriene configurations or a mule alternates between being
a horse and a donkey.

--
Uncle Al
http://www.mazepath.com/uncleal/
http://www.ultra.net.au/~wisby/uncleal/
(Toxic URLs! Unsafe for children and most mammals)
"Quis custodiet ipsos custodes?" The Net!

srp

unread,
May 24, 2001, 12:36:50 PM5/24/01
to
Alan a écrit :

>
> Is there a theory states a photon is a particle when interacting with an
> electron but a wave when not? If there is not, why?

Because, in the 1880-90's, Wien discovered as he experimented with
black body radiation that light did not really behave like waves, as
original Maxwell theory assumed.

Planck studied the case for 10 years and eventually came to the
conclusion that light must in reality be made up of crowds of
innumerable discrete EM-events whose sheer numbers makes light
behave like waves at the macro level, and developped the math
to account for it.

Einstein confirmed the discrete nature of light with the photoelectric
effect, and further confirmations were brought to bear with Compton
and Raman scattering.

These confirmations have never been falsified ever since.

--
André Michaud
Service de Recherche Pédagogique http://www.microtec.net/srp/

Matthew Lybanon

unread,
May 24, 2001, 2:40:26 PM5/24/01
to
Both "wave" and "particle" are abstractions, idealizations. Think of a
(classical) particle, for instance. Something with zero extent but
nonzero mass. Must have infinite density, right? Hmmm, how does that
work? Or think of a wave on a string, or in water. The concept really
refers to a certain type of organized group motion of a collection of
somethings (little volume elements, atoms, however you wish to think of
it on a small scale). Further, there are many examples of what are
clearly (so it would seem) waves exhibiting "particle-like" properties.
Hmmm, if it has energy, momentum, and angular momentum (and is localized
in some way), is it a particle or not?

Fortunately or unfortunately, depending on your point of view, you can't
just say that something IS a particle or IS a wave. The world isn't
made that way.

Vincent Maycock

unread,
May 24, 2001, 9:00:28 PM5/24/01
to

Alan wrote in message <3B0D0FB7...@mac.com>...

>Is there a theory states a photon is a particle when interacting with an
>electron but a wave when not? If there is not, why?

If we have some way of knowing where the photon is, it will behave like a
particle. The only way we have of knowing where a photon is, is to have it
interact with something, though. So in all "interactions," a photon behaves
like a particle. In between interactions, it behaves like a wave. What's
really being wave-like is the probability that the photon will interact with
something at a given point in space.

--
Vince


atilla gurel

unread,
May 25, 2001, 6:43:02 AM5/25/01
to

srp <s...@microtec.net> wrote in message
news:3B0D38A2...@microtec.net...

Discrete nature of the field is not equivalent to the image
of "pointlike or billiardball like particle"

It only means that energy exchange with matter occurs
only as multiples of hw .

Thus there is no "duality" or incompatible complementary images.
There is only field.
And quantized nature is a "property" of field.


Atilla Gurel www.physics-qa.com
Standart QM is already deterministic.
Copenhagen interpretation is only a burden we carry with us

atilla gurel

unread,
May 25, 2001, 6:52:01 AM5/25/01
to

Vincent Maycock <maycock...@cfl.rr.com> wrote in message
news:M8iP6.286318$fs3.48...@typhoon.tampabay.rr.com...

>
> Alan wrote in message <3B0D0FB7...@mac.com>...
> >Is there a theory states a photon is a particle when interacting with an
> >electron but a wave when not? If there is not, why?
>
> If we have some way of knowing where the photon is, it will behave like a
> particle. The only way we have of knowing where a photon is, is to have
it
> interact with something, though. So in all "interactions," a photon
behaves
> like a particle.

That a photon can only absorbed by one single atom and not by several
atoms simultaneously is not an evidence for the existence of a particle
like entity.
Atoms absorbing the photons are themselves also waves. It is only that the
absorbing
entity itself has an average spatial extend of the order *of Angstrom*
that leads to the illusion of photon as "particle" .

> In between interactions, it behaves like a wave. What's
> really being wave-like is the probability that the photon will interact
with
> something at a given point in space.
>
> --
> Vince
>
>

Atilla Gurel www.physics-qa.com

c.h.thompson

unread,
May 25, 2001, 7:24:57 AM5/25/01
to

Uncle Al <Uncl...@hate.spam.net> wrote

> Alan wrote:
> >
> > Is there a theory states a photon is a particle when interacting with an
> > electron but a wave when not? If there is not, why?
>
> Particle and wave are limiting cases of observed reality.

but a "photon" never exists! Waves can interact with electrons.

If "photons" existed how big would they be? How would they get through
glass?

Think of a water wave approaching some ships moored in anchor. It rocks
them, it is partly reflected back, part goes right through and reforms into
much the same wave it started with. Is there any conceivable way that it
could be thought of as a particle? Perhaps all the boats, being identical,
all rock with the same frequency, but this is irrelevant: fixed frequency
does not mean anything much!

Yet when it comes to light we are prepared to forget these basics. Just
because Planck came up with a model that happened to be consistent with the
"photon" idea that Einstein happened to come up with and we all started
saying that photons existed and all had energy hf, we start thinking that
fixed energy (or frequency) implies "particles"!

Caroline

--
c.h.th...@newscientist.net
http://www.aber.ac.uk/~cat


G=EMC^2 Glazier

unread,
May 25, 2001, 8:20:02 AM5/25/01
to
OK We have to get the picture of a particle looking like a golf ball. A
wave in space looking like the waves on top of water. We try looking
into the world of the tiny by smashing things together. That is like
trying to see what is inside a watch by blowing it up. Best we use our
brains,and computers. The photon has in my minds eye parts to it.These
parts can be given off,and later replaced.This we now know the electron
can do.The two slits,or a sharp edge can split them apart,but in a shot
time they will be whole again.It is like all the things that can happen
to a photon traveling through space for say 10 billion LY. It still gets
to the Hubble at the same speed it left its source. Gravity effects its
wave length,and gravity also makes its trip longer,but can't change its
speed. Herb

Richard Herring

unread,
May 25, 2001, 8:45:24 AM5/25/01
to
In article <3b0e4...@news2.vip.uk.com>, c.h.thompson (c.h.th...@newscientist.net) wrote:

> but a "photon" never exists! Waves can interact with electrons.

> If "photons" existed how big would they be? How would they get through
> glass?

Photons are not little billiard balls.

> Think of a water wave approaching some ships moored in anchor. It rocks
> them, it is partly reflected back, part goes right through and reforms into
> much the same wave it started with. Is there any conceivable way that it
> could be thought of as a particle?

Certainly. Never heard of phonons?

> Yet when it comes to light we are prepared to forget these basics. Just
> because Planck came up with a model that happened to be consistent with the
> "photon" idea that Einstein happened to come up with and we all started
> saying that photons existed and all had energy hf, we start thinking that
> fixed energy (or frequency) implies "particles"!

No, we discover that the second quantization of field equations
yields states whose energy is proportional to frequency.

--
Richard Herring | <richard...@baesystems.com>

Spaceman

unread,
May 25, 2001, 9:47:03 AM5/25/01
to
If they got through glass, then they are smaller than glass molocules.
Glass has holes...just so small that air won't go through.

a photon (if that is what creates light.)
is effected by gravity. (eclipses can show this easily.)

a Photon has mass.
and
Of course the mass is too small for anything we have to measure it.
just becasue the scale doesn't read the pea ...does not mean the pea is not
on the scale.
and does not mean the pea does not weigh anything.

Photons have mass.
--
James M Driscoll Jr
Spaceman

If they don't have mass ,
why does light bend around planets.
and please don't give me the Spacetime crap..
Spacetime is measurement and does not make things move
so I don't want to hear that it effect photons because it doesn't.
:)
.


"c.h.thompson" <c.h.th...@newscientist.net> wrote in message
news:3b0e4...@news2.vip.uk.com...

franz heymann

unread,
May 25, 2001, 9:10:45 AM5/25/01
to

atilla gurel <ad...@physics-qa.com> wrote in message
news:3b0e3...@news2.crannews.com...

No. The radius of a photon has been measured in high energy
experiments. It is consistent with being a point particle.

>
> > In between interactions, it behaves like a wave. What's
> > really being wave-like is the probability that the photon
will interact
> with
> > something at a given point in space.

Franz Heymann


franz heymann

unread,
May 25, 2001, 9:07:19 AM5/25/01
to

c.h.thompson <c.h.th...@newscientist.net> wrote in message
news:3b0e4...@news2.vip.uk.com...
>
> Uncle Al <Uncl...@hate.spam.net> wrote
> > Alan wrote:
> > >
> > > Is there a theory states a photon is a particle when
interacting with an
> > > electron but a wave when not? If there is not, why?
> >
> > Particle and wave are limiting cases of observed reality.
>
> but a "photon" never exists! Waves can interact with
electrons.
>
> If "photons" existed how big would they be? How would they get
through
> glass?

The structure factor of photons has been determined by more than
one group of experimenters. It is consistent with zero radius.

>
> Think of a water wave approaching some ships moored in anchor.
It rocks
> them, it is partly reflected back, part goes right through and
reforms into
> much the same wave it started with. Is there any conceivable
way that it
> could be thought of as a particle?

No. That is a classical phenomenon and has no connection with the
probability waves of quantum mechanics, other than that both are
functions of space and time..

> Perhaps all the boats, being identical,
> all rock with the same frequency, but this is irrelevant: fixed
frequency
> does not mean anything much!
>
> Yet when it comes to light we are prepared to forget these
basics.

Your analogy is too useless to allow you to draw any conclusions
from it.

> Just
> because Planck came up with a model that happened to be
consistent with the
> "photon" idea that Einstein happened to come up with and we all
started
> saying that photons existed and all had energy hf, we start
thinking that
> fixed energy (or frequency) implies "particles"!

But you froget that a whole century has passed in the mean time
and that no prediction made by quantum mechanics has ever been
proved wrong in any experimental test.

Franz Heymann

srp

unread,
May 25, 2001, 12:52:49 PM5/25/01
to
atilla gurel a écrit :

>
> srp <s...@microtec.net> wrote in message
> news:3B0D38A2...@microtec.net...
> > Alan a écrit :
> > >
> > > Is there a theory states a photon is a particle when interacting with an
> > > electron but a wave when not? If there is not, why?
> >
> > Because, in the 1880-90's, Wien discovered as he experimented with
> > black body radiation that light did not really behave like waves, as
> > original Maxwell theory assumed.
> >
> > Planck studied the case for 10 years and eventually came to the
> > conclusion that light must in reality be made up of crowds of
> > innumerable discrete EM-events whose sheer numbers makes light
> > behave like waves at the macro level, and developped the math
> > to account for it.
> >
> > Einstein confirmed the discrete nature of light with the photoelectric
> > effect, and further confirmations were brought to bear with Compton
> > and Raman scattering.
> >
> > These confirmations have never been falsified ever since.
> >
> > --
> > André Michaud
> > Service de Recherche Pédagogique http://www.microtec.net/srp/
>
> Discrete nature of the field is not equivalent to the image
> of "pointlike or billiardball like particle"

Discrete EM-events need not be pointlike or billiardball like.

This is the antiquated cm view. We know more now.

Certainly you understand that the physical reality is not
1 for 1 identical to the geometro-mathematical description
that we make of it.

Korzybski said "The map is not the country".



> It only means that energy exchange with matter occurs
> only as multiples of hw .

That's right. All free em-energy seems to be quantized.



> Thus there is no "duality" or incompatible complementary images.
> There is only field.
> And quantized nature is a "property" of field.

I view this as an acausal view, since all we know of free EM-energy
is what occurs when it is emitted (quantized) and when it is absorbed
(unquantized)

A causal view, which de Broglie explains quite simply seems
to imply that quantization of a given quantity of energy
followed by the detection of this exact same quantity precludes
the "merging" into a nondescript underlying field of the initialy
quantized amount of energy between the two events, or else what
could explain that this very precise amount can be precisely
detected at the end of a precisely established trajectory?

"It is obviously certain that the emission of a particle by the
source is the cause of its arrival on the detector. Now, the causal
link between the two phenomena can be established only through the
existence of a trajectory and to deny this existence, is tantamount
to sacrificing causality, it is condemning ourselves not to
comprehend."
Louis de Broglie, 1973

I agree with him. It seems illogical that the initially quantized
amount of energy (photon) would not continuously exist from moment
to moment as it moves along its trajectory, right to the very
moment when it is absorbed by the detector.

srp

unread,
May 25, 2001, 1:09:01 PM5/25/01
to
"c.h.thompson" a écrit :


>
> Yet when it comes to light we are prepared to forget these basics. Just
> because Planck came up with a model that happened to be consistent with the
> "photon" idea that Einstein happened to come up with

Actually, it was the other way around. Planck came up with his model
as he studied Wien's black body experimental results which were at
odds with what Maxwell's wave theory of light seemed to predict (end
of the 1800's.

What Einstein did (1905) (the idea that the photoelectric effect
supported Planck's solution, came after, and consolidated Planck's
conclusions about the black body radiation.

Besides, his E=hf equation does work in all cases.

> and we all started saying that photons existed and all had energy hf,

What he found, and what he published, was that Wien's black body
results were completely explained with hf.

> we start thinking that fixed energy (or frequency) implies
> "particles"!

I found that the word "particle" is used with all kinds of meanings.

Most physicists seem to associate with it the notion of "point-like".
I simply see them as discrete electromagnetic events. Actually,
I see any discrete amount of energy as a particle. I view even
your "wave centers" as "particles".

What is important in my view is that we all ultimately agree
as to what physical "object" we refer to as "particle", beyond
whatever mathematical or geometrical mantel we drape it into.

Do we really know enough to do that at this moment? I have my
doubts.

c.h.thompson

unread,
May 25, 2001, 8:54:38 AM5/25/01
to

atilla gurel <ad...@physics-qa.com> wrote
>
> Vincent Maycock <maycock...@cfl.rr.com> wrote

> >
> > Alan wrote in message <3B0D0FB7...@mac.com>...
> > >Is there a theory states a photon is a particle when interacting
> > >with an electron but a wave when not? If there is not, why?
> >
> > If we have some way of knowing where the photon is, it will
> > behave like a particle. The only way we have of knowing
> > where a photon is, is to have it interact with something, though.
> > So in all "interactions," a photon behaves like a particle.

If you say so, but I think it likely that if we looked hard we'd sometimes
see that same light wave that we thought had been emitted as a single photon
interacting with more than one electron. Would a "particle" do this?

> That a photon can only absorbed by one single atom and

> not by several atoms simultaneously ...

I don't think we even know this! Light is an oscillation in the e/m field.
In other words it is an oscillating electric and magnetic potential. It
will cause many electrons at once to move slightly. The light will not
always induce a change of state. It can alternatively or in addition induce
many slight oscillations of electrons that in turn induce motion in the
neighbouring nuclei and set them oscillating slightly. These oscillations
may be interpreted in various ways, one of which is heat.

> is not an evidence for the existence of a particle like entity.
> Atoms absorbing the photons are themselves also waves.

I agree.

> It is only that the absorbing entity itself has an average spatial
> extend of the order *of Angstrom*
> that leads to the illusion of photon as "particle" .

I'm not sure this is true, and in any case I think the particle illusion has
been exaggerated.

> > In between interactions, it behaves like a wave. What's

> > really being wave-like is the probability ...

Sorry, no go! "Probability" is not a physical thing that can be invoked
like that. It is a derived property of an ensemble of objects but if it
appears to be wave-like that must be because something more fundamental,
some real physical property of the locality, IS behaving as a wave. In the
present instance, Maxwell had it at least approximately right: the e/m field
is waving.

Spaceman

unread,
May 25, 2001, 1:48:39 PM5/25/01
to

"c.h.thompson" <c.h.th...@newscientist.net> wrote in message
news:3b0e9...@news2.vip.uk.com...

> I don't think we even know this! Light is an oscillation in the e/m
field.
> In other words it is an oscillating electric and magnetic potential. It
> will cause many electrons at once to move slightly. The light will not
> always induce a change of state. It can alternatively or in addition
induce
> many slight oscillations of electrons that in turn induce motion in the
> neighbouring nuclei and set them oscillating slightly. These oscillations
> may be interpreted in various ways, one of which is heat.
>

The oscillations are creating the motion of the waves
such as basic vibrations create sound wave motions.
just a much higher vibration rate for light waves.

the orginal oscillations happen at the speed of light or even higher maybe
in
massive explosions such as the Sun
and the wave vibrates and conducts the vibrations at that same speed
until slowed down by something that will stop the osciallation or spread it
out enough to stop .

--
James M Driscoll Jr
Spaceman

www.realspaceman.com

c.h.thompson

unread,
May 25, 2001, 2:28:46 PM5/25/01
to

franz heymann <franz....@care4free.net> wrote

> ... The radius of a photon has been measured in high energy


> experiments. It is consistent with being a point particle.

Then how can it be a wave? And what makes some authors assume that it
cannot get through a hole smaller than one wavelength? (See Ebbesen, T W et
al, "Extraordinary optical transmission through sub-wavelength hole arrays",
Nature 391, 12 February 1998, p667.)

c.h.thompson

unread,
May 25, 2001, 2:48:43 PM5/25/01
to

srp <s...@microtec.net> wrote
>
> ... All free em-energy seems to be quantized.

Where's the evidence? As you say later we only know it's there when we
detect it. Since it has the characteristics of a wave, why should it not
disperse like a wave? I fear you have unwittingly listened to a few
Copenhagists, André!

The secret of the apparently paradoxical behaviour of light is in the way we
detect it, using instruments that cannot register fractions of a photon.
Alternatively, one might argue that what is detected is partly energy from
the official source but partly general background energy -- zero point
field. The sum adds to one "quantum".

But personally I think Nature does quite a lot of operations without
bothering to build up to the neat quantum units. When push comes to shove,
She realises that what light is is merely an oscillation in the e/m field,
and this causes unquantised motions in all charged particles it encounters.

> ... all we know of free EM-energy is what occurs


> when it is emitted (quantized) and when it is absorbed
> (unquantized)

This sounds OK. Did I misjudge you?

> A causal view, which de Broglie explains quite
> simply seems to imply that quantization of a
> given quantity of energy followed by the detection
> of this exact same quantity

That's where I think you've been misled, Andé. I don't think it is ALWAYS
exactly the same quantity, and even if it were it could be that local energy
is used to bring the signal up to standard level.

> ... what could explain that this very precise amount


> can be precisely detected at the end of a precisely
> established trajectory?

There are crucial gaps in the evidence for this. When light arrives at a
detector, how can you be sure it is ONLY light from the "source" that you
are detecting?

> "It is obviously certain that the emission of a particle by the
> source is the cause of its arrival on the detector. Now, the
> causal link between the two phenomena can be established
> only through the existence of a trajectory and to deny this
> existence, is tantamount to sacrificing causality, it is
> condemning ourselves not to comprehend."
> Louis de Broglie, 1973

But light is not a particle!

> I agree with him. It seems illogical that the initially
> quantized amount of energy (photon) would not
> continuously exist from moment to moment as it
> moves along its trajectory, right to the very
> moment when it is absorbed by the detector.

The WAVE continuously exists ...

--
c.h.th...@newscientist.net
http://www.aber.ac.uk/~cat

c.h.thompson

unread,
May 25, 2001, 3:11:17 PM5/25/01
to

Richard Herring <r...@gmrc.gecm.com> wrote

> c.h.thompson (c.h.th...@newscientist.net) wrote:
>
> > but a "photon" never exists! Waves can interact with electrons.
>
> > If "photons" existed how big would they be? How would
> > they get through glass?
>
> Photons are not little billiard balls.
>
> > Think of a water wave approaching some ships moored in
> > anchor. It rocks them, it is partly reflected back, part goes
> > right through and reforms into much the same wave it started
> > with. Is there any conceivable way that it
> > could be thought of as a particle?
>
> Certainly. Never heard of phonons?

I should have said "any REASONABLE way!". As Willis Lamb says in that
article (Lamb, Willis E Jr., "Antiphoton", Applied Physics B 60, 77-84
(1995), phonons had no more right to be invented than photons. I know Franz
says that phonons have their uses, but I'm unconvinved.

[skip my stuff on Planck and Einstein]

> ... we discover that the second quantization of field equations


> yields states whose energy is proportional to frequency.

Do I detect a certain circularity in the development of the theory?

franz heymann

unread,
May 25, 2001, 4:18:54 PM5/25/01
to

c.h.thompson <c.h.th...@newscientist.net> wrote in message
news:3b0eb0d2$1...@news2.vip.uk.com...

>
> Richard Herring <r...@gmrc.gecm.com> wrote
> > c.h.thompson (c.h.th...@newscientist.net) wrote:
> >
> > > but a "photon" never exists! Waves can interact with
electrons.
> >
> > > If "photons" existed how big would they be? How would
> > > they get through glass?
> >
> > Photons are not little billiard balls.
> >
> > > Think of a water wave approaching some ships moored in
> > > anchor. It rocks them, it is partly reflected back, part
goes
> > > right through and reforms into much the same wave it
started
> > > with. Is there any conceivable way that it
> > > could be thought of as a particle?
> >
> > Certainly. Never heard of phonons?
>
> I should have said "any REASONABLE way!". As Willis Lamb says
in that
> article (Lamb, Willis E Jr., "Antiphoton", Applied Physics B
60, 77-84
> (1995), phonons had no more right to be invented than photons.
I know Franz
> says that phonons have their uses, but I'm unconvinved.

Unless you read up some solid state physics, I am afraid you are
doomed to remain unconvinced, entirely to your own disadvantage.

>
> [skip my stuff on Planck and Einstein]
>
> > ... we discover that the second quantization of field
equations
> > yields states whose energy is proportional to frequency.
>
> Do I detect a certain circularity in the development of the
theory?

You might, but you would be wrong to do so.
Why don't you learn some QED and disabuse yourself of this
misunderstanding?

Franz Heymann


franz heymann

unread,
May 25, 2001, 4:51:59 PM5/25/01
to

c.h.thompson <c.h.th...@newscientist.net> wrote in message
news:3b0eb0d0$1...@news2.vip.uk.com...

>
> franz heymann <franz....@care4free.net> wrote
>
> > ... The radius of a photon has been measured in high energy
> > experiments. It is consistent with being a point particle.
>
> Then how can it be a wave?

If you femiliarised yourself with QED you would understand this.
But briefly, the wave function is a concept which is used to
determine
probabilities of obtaining certain results when observing a
photon, like its position, its energy or its spin direction.
The wavelength has nothing to do with the possible
physical size of a photon.

> And what makes some authors assume that it
> cannot get through a hole smaller than one wavelength? (See
Ebbesen, T W et
> al, "Extraordinary optical transmission through sub-wavelength
hole arrays",
> Nature 391, 12 February 1998, p667.)

I have not read this paper, so I won't comment except by saying
that the diffraction of a wave by a small hole is calculable and
hence the probability that a photon would appear on the opposite
side of the hole is calculable. The fact that a photon can
traverse a hole smaller than its wavelength is a crude indication
that the size of a photon and its wavelength are not necessarilly
connected.
The calculation to which I refer was originally done more than a
century ago by Rayleigh, long before the advent of quantum
theory. The same calculations are still applicable, as long as
proper care is taken with the normalisation of the wave, to
ensure that the total probability of finding the photon somewhere
is 1.

Franz Heymann

srp

unread,
May 25, 2001, 6:32:06 PM5/25/01
to
"c.h.thompson" a écrit :

>
> srp <s...@microtec.net> wrote
> >
> > ... All free em-energy seems to be quantized.
>
> Where's the evidence?

The evidence that convinced me is three-fold.

Photoelectric effect, Compton scattering and Raman scattering.

Studying those three and correlating the info thus verified
swayed me about 90% towards de Broglie's particle view.

Crookes tube study showed me conclusively that it can be nothing
but Coulomb induced unquantized quantities of motion that can
be actually quantized.

Understanding how de Broglie's dual particle photon concept
can be reconciled with Maxwell, as he says, did the rest.

I am now convinced, because it all makes global sense, and the
causality thread is unbroken to my satisfaction.

> As you say later we only know it's there when we
> detect it.

Yes. All we can directly verify is what we can detect. The rest of
what we know can only be deduced from that.

> Since it has the characteristics of a wave, why should it not
> disperse like a wave?

This is the crux of the matter. At the fundamental level, quantized
em-events do not have the characteristics of a wave. This is what
Wien discovered as he studied his black body data and that Planck
explained.

> I fear you have unwittingly listened to a few Copenhagists,
> André!

If you check with the few obnoxious copenhagists who have been
constantly on my case here for years, you will learn from them
that I listen to absolutely no one. I took only evidence that
was verified to my satisfaction and made up my own mind.

Open to reassessment at any time if any new piece of evidence
to the contrary is ever verified to my satisfaction.

I studied the evidence from as many sources as I could get a hold
of. I listened to arguments from all sides, including those of
the long gone causalists, Faraday, Maxwell, Poincaré, Planck,
de Broglie that no one has listened to for half a century, and got
on the trail I now follow long before I came online here.

I trust only experimental evidence whose validity has been
verified to my satisfaction.



> The secret of the apparently paradoxical behaviour of light is in the
> way we detect it, using instruments that cannot register fractions of
> a photon.

From my understanding, the behavior seems paradoxical only because we
tend to not clearly distinguish macro level observation with elementary
level behavior.

I tried to discard all macro level conclusion that seemed to me to be
extranuous.

> Alternatively, one might argue that what is detected is partly energy
> from the official source but partly general background energy -- zero
> point field. The sum adds to one "quantum".

This can certainly be hypothesized, but how could evidence for this
be verified?

> But personally I think Nature does quite a lot of operations without
> bothering to build up to the neat quantum units.

From what I understand, quantum units of any fractional amount can
exist. All possible cases of photon emission by electrons going down
to lower orbitals in all possible types of atoms, in any possible
ambiant energy circumstances, and all possible cases of quarks up
and down going down to lower states of energy account for the whole
possible physically existing range.

> When push comes to shove,
> She realises that what light is is merely an oscillation in the e/m field,
> and this causes unquantised motions in all charged particles it encounters.

It certainly may look like that as seen from the macro level.



> > ... all we know of free EM-energy is what occurs
> > when it is emitted (quantized) and when it is absorbed
> > (unquantized)
>
> This sounds OK. Did I misjudge you?

Did you :o]



> > A causal view, which de Broglie explains quite
> > simply seems to imply that quantization of a
> > given quantity of energy followed by the detection
> > of this exact same quantity
>
> That's where I think you've been misled, Andé. I don't think it is
> ALWAYS exactly the same quantity,

In nature certainly not, because the relative motion of the emitter
and the receiver must be taken into account. But in controled
experiments, it has been explained to my satisfaction that photons
of a given energy can be reliably detected at the very same level
of energy for emitter and receiver immobile with respect to each
other.

> and even if it were it could be that local energy is used to bring
> the signal up to standard level.

Again, theoretically possible, but how could such a situation be
verified.

I came to trust that the fundamental behavior of em energy is
simple and follow simple predicatable laws.



> > ... what could explain that this very precise amount
> > can be precisely detected at the end of a precisely
> > established trajectory?
>
> There are crucial gaps in the evidence for this.

We will never, even in a million years, have more evidence than
this. This is it. The connection to be made between emission and
detection has to be made logically.

Causalists like Planck, de Broglie, see a break in the logical
thread of causality if the continued existence of the emitted
quantized quantity is not assumed a physical reality.

I agree with them. All other options seem illogical to me.

> When light arrives
> at a detector, how can you be sure it is ONLY light from the "source"
> that you are detecting?

Well, to my knowledge, it has been possible for decaces to set up
experiments where photons are emitted individually in a verifiable
manner, with orientation and shielding set up to garantee that no other
photon interferes, and that this photon is then detected individually.

Or have I misunderstood something fundamental?



> > "It is obviously certain that the emission of a particle by the
> > source is the cause of its arrival on the detector. Now, the
> > causal link between the two phenomena can be established
> > only through the existence of a trajectory and to deny this
> > existence, is tantamount to sacrificing causality, it is
> > condemning ourselves not to comprehend."
> > Louis de Broglie, 1973
>
> But light is not a particle!

I know that this is your opinion. But the evidence plus the sense
to be made of de Broglie's theory of the dual-particle photon
has definitely convinced me otherwise.



> > I agree with him. It seems illogical that the initially
> > quantized amount of energy (photon) would not
> > continuously exist from moment to moment as it
> > moves along its trajectory, right to the very
> > moment when it is absorbed by the detector.
>
> The WAVE continuously exists ...

That's what Maxwell and Faraday concluded, but since Wien's
experiment, evidence to the contrary seems sufficient to
make the particle view a real option.

A problem to finally settled when we have the final solution.

Actually, I make no attempts at convincing anyone. I just air
my opinion on this.

People cannot be convinced. They must look at the evidence
and make up their own individual minds.

c.h.thompson

unread,
May 26, 2001, 5:08:09 AM5/26/01
to

franz heymann <franz....@care4free.net> wrote
>
> c.h.thompson <c.h.th...@newscientist.net> wrote

> >
> > franz heymann <franz....@care4free.net> wrote
> >
> > > ... The radius of a photon has been measured in high energy
> > > experiments. It is consistent with being a point particle.
> >
> > Then how can it be a wave?
>
> If you familiarised yourself with QED you would understand

> this. But briefly, the wave function is a concept which is used
> to determine probabilities of obtaining certain results when
> observing a photon, like its position, its energy or its spin direction.
> The wavelength has nothing to do with the possible
> physical size of a photon.

But see later ...

> > And what makes some authors assume that it
> > cannot get through a hole smaller than one wavelength?
> > (See Ebbesen, T W et al, "Extraordinary optical
> > transmission through sub-wavelength hole arrays",
> > Nature 391, 12 February 1998, p667.)
>
> I have not read this paper, so I won't comment except by saying
> that the diffraction of a wave by a small hole is calculable and
> hence the probability that a photon would appear on the opposite
> side of the hole is calculable. The fact that a photon can
> traverse a hole smaller than its wavelength is a crude indication
> that the size of a photon and its wavelength are not necessarilly
> connected.

I'm glad you included the words "not necessarily"! Does this mean that the
standard QM argument about how two-slit interference patterns are formed
cannot necessarily be applied elsewhere? How do you know when the wave
function DOES tell you the wavelength and when it doesn't?

> The calculation to which I refer was originally done more than
> a century ago by Rayleigh, long before the advent of quantum
> theory. The same calculations are still applicable, as long as
> proper care is taken with the normalisation of the wave, to
> ensure that the total probability of finding the photon
> somewhere is 1.

I would humbly submit that Rayleigh's work would be best left unnormalised
as there is no logical reason to normalise it. The idea that the "photon"
will be found in precisely one place is a myth. As if this did not follow
logically from the fact that it is a wave, the "single-photon" interference
experiments could be used to show that it is nonsense! All you need is
several detectors, set at expected peaks ...

Incidentally, isn't this about what Pfleegor and Mandel did when they did
their very interesting demonstration of interference between "photons" from
different sources? See Pfleegor, R L and Mandel L, "Interference of
Independent Photon Beams", Physical Review 159, 1084 (1967).

franz heymann

unread,
May 26, 2001, 6:49:52 AM5/26/01
to

c.h.thompson <c.h.th...@newscientist.net> wrote in message
news:3b0f7...@news2.vip.uk.com...

There is every reason in the world to normalise the wavefunction
of a photon. It is part of the definition of a wave function. I
fear this is yet another instance of your not knowing what you
are speaking about.

> The idea that the "photon"
> will be found in precisely one place is a myth.

Then please explode the myth by telling us of a single instance
in which a photon has been found to be in two places at once.

> As if this did not follow
> logically from the fact that it is a wave, the "single-photon"
interference
> experiments could be used to show that it is nonsense! All you
need is
> several detectors, set at expected peaks.

I take it that you are aware of a calculation to determine at
which two places the detectors should be placed, or that you have
done such an experiment, or seen it done? If not, I have to
assume that this is another instance of blind handwaving.

>
> Incidentally, isn't this about what Pfleegor and Mandel did
when they did
> their very interesting demonstration of interference between
"photons" from
> different sources? See Pfleegor, R L and Mandel L,
"Interference of
> Independent Photon Beams", Physical Review 159, 1084 (1967).

No it is not. You are diverting attention from the issue under
discussion.

Franz Heymann

franz heymann

unread,
May 26, 2001, 9:24:25 AM5/26/01
to

c.h.thompson <c.h.th...@newscientist.net> wrote in message
news:3b0f7...@news2.vip.uk.com...
> I'm glad you included the word "not necessarily"!

If it makes you happy, I am contented to leave out the word
"necessarily". My statement will still be correct.

> Does this mean that the
> standard QM argument about how two-slit interference patterns
are formed
> cannot necessarily be applied elsewhere?

"Elsewhere" requires a different QM argument, appropriate to the
circumstances. Particles do not produce the same diffraction
pattern when they pass through a hole or through a single slit as
that which is produced when they are involved in a two-slit
situation.


> How do you know when the wave
> function DOES tell you the wavelength and when it doesn't?

Always, if the wave function which describes the object is either
that of an eigenstate of momentum, or if it consists of a number
of superposed momentum eigenstates all with the same magnitude of
the momentum (hint: a stationary wave). In other circumstances
it does not have the attribute of a wavelength.

Aleksandr Timofeev

unread,
May 26, 2001, 12:15:43 PM5/26/01
to
"franz heymann" <franz....@care4free.net> wrote in message news:<3b0e6427$0$12246$cc9e...@news.dial.pipex.com>...
[snip]

1.


> The structure factor of photons has been determined by more than
> one group of experimenters. It is consistent with zero radius.

[snip]

2.


> But you froget that a whole century has passed in the mean time
> and that no prediction made by quantum mechanics has ever been
> proved wrong in any experimental test.
>
> Franz Heymann

You can play with a virtual radio interferometer.

If you will manage to explain a principle of operation of this type
of interferometer from the photon point of view, then:

I shall believe in existence of photons, and I shall eat my tie or hat.
---------------------------------------

I think, that your navel will be untied, but you can not explain
----------------------------------------------------------------
a principle of operation of a virtual interferometer from a photon
point of view!


Give an evaluation to the size of a photon if the photon has
a wavelength 3.5 cm. and if the distance between antennas is equal
to diameter of globe. :O) !

Any attempts to explain the principle of operation of the given
type of the radio interferometer from a photon point of view
will suffer a fall.

The radio interferometer with independent writing of signals on
"slots" is a direct proof of a non-existence of photons in a nature.

By my former scientific chief Matveenko Leonid Ivanovich (he works
at the Space Research Institute RAS till now) in 1963 was invented
the absolutely new kind of the radio interferometer.

Main ideas of this type of the radio interferometer were:
1. A simultaneous independent recording of signals on each
separate antenna ("slot") on magnetic tapes;
2. " The interference picture " is received in the computer
as an outcome _ mathematical _ addings of signals recorded on
magnetic tapes;
3. The distance between antennas ("slots ") of a radio
interferometer can exceed diameter of the Earth. (For definit
wave length limiting distances between antennas, at which the
interference picture disappears, is not known until now!)

There are no problems for explanation of a principle of operation
of the radio interferometer with simultaneous independent writing
of signals from a wave point of view.

---------------------
Here for the first time clearly emerges, that for a hypothetical
particle of a photon there is no necessity to pass simultaneously
through both slots (antennas), since the virtual interference
abstractly or mathematically will be realized in the computer at
any convenient time hereafter. !!! It is the experimental fact!!!

How the admirers of a hypothesis of photons now will explain
an interference?
---------------------

But any attempts to explain a principle of operation of the given
type of the radio interferometer from a photon point of view will
suffer a fall for the following reasons:

1. In this case there is no real physical process of an
interference - interference will be realized abstractly
mathematically in the computer;
2. Give an evaluation to the size of a photon if the photon has
a wavelength 3.5 cm. and if the distance between antennas is equal
to diameter of globe. Average on space density of energy impresses,
the delay of time in all processes impress too.


The quantum microsystems can absorb energy only by quantum
portions. This energy is absorbed as electromagnetic waves by
quantum microsystems at random coincidence of orientation of a
spatial dynamic configuration of a quantum microsystem with
orientation of an electromagnetic wave. Analogy between a quantum
microsystem and directional antenna here is conducted in an
obvious kind.

-------------------------------------------------------------------------
From: Aleksandr Timofeev (t...@alpha.dnttm.rssi.ru)
Subject: PHOTONS DO NOT EXIST IN A NATURE
Newsgroups: sci.physics.particle
Date: 1999/01/19

On my sight, for consideration of an offered problem from all points of
view, the most approaching measuring instrument is the microwave interferometr
with superlong basis. I would name this type of an interferometer as an
interferometer with independent registration of signals in shoulders.
Fundamentally any other interferometer by nothing differs from
an interferometer considered below.

Principles of work.

The microwave interferometr with superlong basis consists of two radio
telescopes were on a very large distance from each other. Before experiment
or after him, the nuclear hours are synchronized. Each radio telescope writes
on a videotape a transformed radiation accepted by an antenna. Simultaneously
with a signal, the scores of time received from the standard of frequency,
are written on a videotape.
After ending experiment we have two videotapes with entries of a signal
and scores of time. The "interference picture" is received after data
processing of these videotapes on the computer.

There are two graphic schemes illustrating the description:

The microwave interferometer with superlong basis. Part 1.
Block scheme.

-> radio-telescope 1
->
-> parabolic antenna tape 1 clock 1
-> \
-> \ [ microwave ]
-> \ [ receiver + ] [videotape] [hydrogen ]
-> ) )--->[analog-to-digital]--->[recorder ]<---[frequency]
-> / [ converter ] ^ ^ [standard ]
-> / | |
-> / radio-signals time-marks
-> microwave
-> radiation
-> for synchronization of atomic clocks
-> [transportable caesium]
-> [ frequency standard ]
[snip] ====================================================================
-> radio-telescope 2
->
->
-> parabolic antenna 2 tape 2 clock 2
-> \
-> \ [ microwave ]
-> \ [ receiver + ] [videotape] [hydrogen ]
-> ) )--->[analog-to-digital]--->[recorder ]<---[frequency]
-> / [ converter ] ^ ^ [standard ]
-> / | |
-> / radio-signals time-marks
->
->
->
The microwave interferometer with superlong basis. Part 2.
----------------------------------------------------------

"Interference picture"
^
|
[videotape 1] ------> [ COMPUTER ] <---------- [videotape 2]
^ ^
| |
radio-telescope 1 <- synchronization clocks -> radio-telescope 2
Length of basis
|<----------------------------- {snip} ------------------------------->|
/^\ /^\

^ ^ ^ ^ ^ ^ ^ ^ ^ ^ ^ ^ {snip} ^ ^ ^ ^ ^ ^ ^ ^ ^ ^ ^
| | | | | | | | | | | | ...... | | | | | | | | | | |
Noise microwave radiation


Flexible possibilities of a computer interference of signals.

1. Our interferometer has the right and left shoulders. The distance
between shoulders does not influence sensitivity of an interferometer. The
sensitivity of an interferometer to a signal is determined by the worse
receiver from both radio telescopes.
- The distance between shoulders of an interferometer can be no matter
how large. (This problem is reduced to a problem of transportation of clocks
of a synchronization).
2. The addition of signals is carried out in the computer, that allows
to apply no matter how complicated algorithms of addition of signals.
- In that specific case, we can arbitrary vary delay of signals in each
from a shoulders in any direction.

Conditionality of physical concept " an Interference picture ".

Here we shall be convinced of a celebration of a principle of a causality.
The events happening on slots of an interferometer have primary significance,
all other events happening in an interferometer have the status
secondary.
Let's analyze physical concept addition of signals in an interferometer.
The radiation incident on an input of an interferometer has the following
characteristics:
Wavefront; Frequency band; Spectral fluence of energy;
For each frequency:
Polarization; Amplitude; Phase; Stability.
The interferometer considered by us, is an interferometer with
independent registration of signals in shoulders and the process of addition
of signals is carried out in the computer. The phrase " process of addition
of signals is carried out in the computer " allows clearly to seize essence "
concepts of an interference picture " and source of an origin of this
concept. In the given type of an interferometer there is some arbitrariness
in choice by us of the law of addition of signals from the right and left
shoulders. In our case " the kind of an interference picture " depends on
the concrete law of addition of signals selected by us. In other kinds of
interferometers geometry (physical) construction of an interferometer
determines the law of addition of signals and " a kind of an interference
picture ".
Conclusion 1. For existence of the phenomenon of an interference the
first necessary condition is the registration of a signal for two sites (two
orifices) of a wavefront, which are located on some distance from each other.
Conclusion 2. For existence of the phenomenon of an interference the
second necessary condition is the filtration of frequency band of accepted
signals. The band of a signal should decrease with a diminution of a distance
between two sites (radiation flux) of a wavefront and on the contrary. The
stability of a source of a signal plays the role also.
Conclusion 3. For existence of the phenomenon of an interference the
third necessary condition is the sufficient total energy of an accepted signal
or it is necessary to accumulate a signal.
Conclusion 4. For existence of the phenomenon of an interference the fourth
necessary condition is the existence of the adder of signals from the right
and left shoulders.

Major parameters of an interferometer.

Give is summable stated above. Major parameters of an interferometer are:
Distance between antennas (slots);
Frequency band of registered signals;
Sensitivity of an interferometer.


Sensitivity of an interferometer.

The interferometer represents the macroscopic system.
What determines sensitivity of an interferometer? The effective square of
an antenna determines a stream of energy of a signal, than more square of an
antenna by that the greater stream of energy acts on an input of the
amplifier of a signal. The first cascade of the amplifier of a signal
determines quality of the amplifier. It represents the macroscopic system. If
the macroscopic system is in a thermodynamic equilibrium, it's the state is
characterized in temperature.
This temperature characterizes own noise of the amplifier. Energia of a
signal should exceed energy of own noise of the amplifier, therefore own
noise of the amplifier set threshold energy of a signal, which determines
sensitivity of an interferometer.
The minimum energy stream discovered by an interferometer is close
to 3*10^-21 wt/m^2, that is equivalent to several tens photons on a
square meter per one second (this value requires multiplication on square of
an antenna).

Conclusions.

1. In a considered microwave interferometer, the properties of a rather
sparse stream of absorbed photons are registered. (What is the time of life of
"photon" absorbed by an antenna?). Interferometer has "a very large mass".
2. Varying delay of time of a signal in the certain shoulder of an
interferometer, we define stability of frequency of a source in a direction
of a falling stream. The similar experiments frequently were carried out in
optics.
3. A new information we could receive increasing a distance between
radio telescopes. We could clarify on what distance on a wavefront the
correlation disappears (if disappears). The similar experience are not known
to me. For basis equal to the Earth's diameter, the correlation reliably
exists. The resolving power of the radio interferometer approximately in
100 - 1000 times is exceeded with a resolving power of an optical telescope.
4. We shall name photons, in a stream incident on an antenna, " as free
photons ". Let's name photons absorbed by an antenna, " as bound photons ".
Whether we know a structure " of bound photons "? To me it is not clear. I
definitely know, that the structure of an electromagnetic field is determined
by boundary conditions. For this reason the structure " of free photons
" should differ from a structure " of bound photons ". The detailses of
mechanisms of a radiation and absorption of an electromagnetic field are not
known to us. On my sight the photon is simply other title for mechanisms of
a radiation and absorption of an electromagnetic field. For this reason the
PHOTONS DO NOT EXIST IN A NATURE or, if it is pleasant more to you, in
a medium of an electromagnetic field the photons will be generated as virtual
particles.
We shall come to a conclusion:
The photons are particles - ghosts, the photons are mathematical
abstraction, which allow us to calculate probability of interaction of an
electromagnetic field and substance.
-------------------------------------------------------------------------

franz heymann

unread,
May 26, 2001, 2:19:17 PM5/26/01
to

Aleksandr Timofeev <a_n_ti...@my-deja.com> wrote in message
news:e16a4a22.01052...@posting.google.com...

> "franz heymann" <franz....@care4free.net> wrote in message
news:<3b0e6427$0$12246$cc9e...@news.dial.pipex.com>...
> [snip]
>
> 1.
> > The structure factor of photons has been determined by more
than
> > one group of experimenters. It is consistent with zero
radius.
>
> [snip]
>
> 2.
> > But you froget that a whole century has passed in the mean
time
> > and that no prediction made by quantum mechanics has ever
been
> > proved wrong in any experimental test.
> >
> > Franz Heymann
>
> You can play with a virtual radio interferometer.
>
> If you will manage to explain a principle of operation of this
type
> of interferometer from the photon point of view, then:
>
> I shall believe in existence of photons, and I shall eat my tie
or hat.
> ---------------------------------------

[Snip]

You used too many words in a very repetitve way, so I snipped all
of them
I hope you have either a hat or a tie to eat, because here is the
explanation to your problem :-

Firstly, the wave has nothing to do with the size of a photon.
The wave function allows one to calculate various probabilities
associated with the detection of a photon.

Secondly, it is an observable fact that the wave function of
radio frequency photons emitted by an astronomical object can
have a non zero amplitude at any of the detectors of a large
baseline interferometer. In fact, it expands radially outwards
from the source as a spherical wave.

Thirdly, photons are bosons, which means that any number of them
can share the same wavefunction.

Fourthly, a macroscopic radio wave is the wavefunction of an
extremely large assembly of photons, all coherently sharing the
same wave function. Detecting one photon at one of the antennae
simultaneously with the detection of another (coherent) photon at
another antenna is then possible.

Fifthly, in the classical limit, the behaviour of such a coherent
assembly of photons has been proven to tend to just the behaviour
one would expect from an application of Maxwell's equations.

Now get the frying pan ready and prepare your tie or hat for
eating.

Bon appetit.

Franz Heymann


Charles Francis

unread,
May 27, 2001, 3:45:28 AM5/27/01
to
In article <3b0eb0d0$1...@news2.vip.uk.com>, c.h.thompson <c.h.thompson@n
ewscientist.net> writes

>
>franz heymann <franz....@care4free.net> wrote
>
>> ... The radius of a photon has been measured in high energy
>> experiments. It is consistent with being a point particle.
>
>Then how can it be a wave?

It can't. The wave is only used for the calculation of probability. It
is a complex valued function and as such no more represents physical
reality than the square root of minus one. A calculational device, that
is all.

>And what makes some authors assume that it
>cannot get through a hole smaller than one wavelength? (See Ebbesen, T W et
>al, "Extraordinary optical transmission through sub-wavelength hole arrays",
>Nature 391, 12 February 1998, p667.)

It they really assume that, the answer is simple. They don't know what
they are talking about.

--
Charles Francis

Charles Francis

unread,
May 27, 2001, 3:59:47 AM5/27/01
to
In article <3b0eb0d1$1...@news2.vip.uk.com>, c.h.thompson <c.h.thompson@n
ewscientist.net> writes

> Since it has the characteristics of a wave, why should it not
>disperse like a wave?

If you do not know where a particle is going, you need a formalism that
describes the possibility that it goes in different direction. Only the
formalism behaves like waves. It is always a particle when it is
confined. Of course it never is perfectly combined, and nor do we know
precisely the mechanisms in the apparatus used to confine it. So this
lack of knowledge also requires a wave in the formalism. You continue to
confuse our lack of complete knowledge with reality. Waves only describe
our lack of knowledge.
--
Charles Francis

srp

unread,
May 27, 2001, 8:07:14 AM5/27/01
to
Charles Francis a écrit :

I could not have said it better.

The one suggestion I can offer, Caroline, would be for you to
have a personal look at the evidence that convinced others of
the discreteness of em-energy at the fundamental level, starting
with the event that started it all, which is Wien's black body
experiment, and follow the trail from that point on.

That's actually what I did at some point, when I got tired of all
the contradictory arguments from all sides. I am afraid that we
could otherwise argue the point until kingdom comes without
resolution.

Zundark

unread,
May 27, 2001, 1:21:56 PM5/27/01
to
Charles Francis wrote:

> The wave is only used for the calculation of probability. It
> is a complex valued function and as such no more represents physical
> reality than the square root of minus one. A calculational device,
> that is all.

Why do you assume that a complex-valued function cannot represent
physical reality?

--
Zundark

Spaceman

unread,
May 27, 2001, 2:48:58 PM5/27/01
to
You can assume it does,
but it does not mean it is.

I can assume a picture is a horse,
but it aint no horse itself.
only a picture of the horse.

"Zundark" <zund...@alberteinstein.co.uk> wrote in message
news:MPG.157b561c9...@news.freeserve.net...

Charles Francis

unread,
May 28, 2001, 3:55:47 AM5/28/01
to
In article <MPG.157b561c9...@news.freeserve.net>, Zundark
<zund...@alberteinstein.co.uk> writes

Mathematics, in its inception, was drawn out of the properties of nature
by abstraction. We abstract 3+4=7 from three apples plus four apples
equals seven apples. But mathematics goes beyond abstraction. We also
use construction and extension in mathematics, so we can consider formal
rules applied to objects of our thought, using only self consistency as
a constraint, knowing that when we restrict this extended structure to
statements about reality then such statements will be true if the
assumptions are true.

The scientific use of mathematics in physics means that we must have an
empirical basis for it. We have an empirical basis for integer numbers.
But we have no empirical basis for complex numbers. If it comes to it we
do not even have an empirical basis for real numbers, and any claim that
they exist in nature is as scientific as the theory of phlogiston.


--
Charles Francis

Aleksandr Timofeev

unread,
May 29, 2001, 6:34:59 AM5/29/01
to
"franz heymann" <franz....@care4free.net> wrote in message news:<3b0ff3b5$0$15026$cc9e...@news.dial.pipex.com>...

I doubt of that that I should make it absolutely
and in essence. :-(

> because here is the explanation to your problem :-
>
> Firstly, the wave has nothing to do with the size of a photon.
> The wave function allows one to calculate various probabilities
> associated with the detection of a photon.
>
> Secondly, it is an observable fact that the wave function of
> radio frequency photons emitted by an astronomical object can
> have a non zero amplitude at any of the detectors of a large
> baseline interferometer. In fact, it expands radially outwards
> from the source as a spherical wave.
>
> Thirdly, photons are bosons, which means that any number of them
> can share the same wavefunction.
>
> Fourthly, a macroscopic radio wave is the wavefunction of an
> extremely large assembly of photons, all coherently sharing the
> same wave function. Detecting one photon at one of the antennae
> simultaneously with the detection of another (coherent) photon at
> another antenna is then possible.
>
> Fifthly, in the classical limit, the behaviour of such a coherent
> assembly of photons has been proven to tend to just the behaviour
> one would expect from an application of Maxwell's equations.
>

Here we have introduction of a hypothesis " of a flow of
coherent photons ".

The virtual interference picture is saved by hours in the
computer, for this reason yours " the chimerical flow of photons "
should be assimilated to a soldier's regime. This soldier's regime
should be of a vast expansion in space, since the radius of area
of coherent space should be equal to a product, for example,
10 hours on speed of light...

This picture physically is inconsistent also, this one there
is simplly physical delirium.
All by you written here is attempt of introduction of
artificial additional hypothesises for saving an absolutely
excessive hypothesis of a photon.

Other scientifically fancy fairy tale about an interference
of photons on two slots is adduced in the book:

Richard Feynman "THE CHARACTER OF PHISICAL LAW";
A series of lectures recorded by the BBC at Cornell University USA;
Cox and Wynman LTD, London, 1965

this fairy tale is refuted by experimental existence of a virtual
interference. ;o)

Your description was reduced to the classical wave description
of an appearance of an interference. !!!!!!! ;o)

Your representation about a wave function is inconsistent also:

"Thus I'm reluctant to talk about the issues you're raising now. They're
too fancy for this conversation. I'll just whisper to you the approach
I'm implicitly taking towards this question:
" John Baez

---------------------------------------------------------------------
From: john baez (ba...@galaxy.ucr.edu)
Subject: Re: photon wave-functions?
Newsgroups: sci.physics.research
Date: 1999/01/27

In article <78d7ds$q...@news.dtc.hp.com>,
(Greg Weeks) <we...@orpheus.dtc.hp.com> wrote:
>In the discussion single-photon wavetrains, it seems to be generally
>assumed that the photon has a wave-function. Even in free field theory, I
>don't believe this is true.

Education is a process of telling a carefully chosen sequence of lies
in which the amount of deliberate deception gradually tends towards zero.
There is a limit to how much truth someone can absorb all at once without
their brain turning to jelly!

Oz - or whoever originally asked the question - seems to be wondering
something like "what's the shape of the wavefunction of a photon of a
given energy?" Of course they're not phrasing it that way, but that's
my desperate attempt to translate it into something I can understand.

Now you're right, it's a bit of a pity that they chose a *photon* as
the particle to ask about in this question. Massless particles are a
nuisance because the Newton-Wigner localization breaks down. Gauge
bosons are a nuisance because it's harder to separate out the physical
degrees of freedom in a gauge theory. So even *ignoring* the extra
subtleties when we take interactions into account and drop the pleasant
fictions of free field theory and Fock space, we have some serious
issues to deal with in a complete answer to this question!

But if someone asks the question "what's the shape of the wavefunction
of a photon of a given energy?" and you start talking to them about
Newton-Wigner localization, gauge-invariance, and Fock space, their
brain is going to turn to jelly! They're going to walk away in a daze
having learned nothing. They'll probably be shocked that such a simple
question elicited such a complicated bunch of mumbo-jumbo. They may
become politicians and cut funding for physics.

So you have to tell them something helpful even if it's oversimplified.

First and foremost, it seems to me, you have to disabuse of them of the
assumption that the wavefunction of a particle has some fixed "wavetrain
with finitely many wiggles" shape that depends solely on the energy of the
particle. When one starts out learning physics, one tends to think of
a particle as a little tennis ball or something, perhaps with some wiggly
waves thrown in for good measure. The idea that it's just a "field mode"
doesn't come easily! Usually one absorbs this slowly and painfully by
solving Schrodinger's equation with all sorts of different boundary conditions
and potentials, learning all sorts of different orthonormal bases for the
space of states, and eventually realizing that the choice of basis is just
a matter of convenience. The idea that a particle is just a solution of a
partial differential equation and that there are *lots* of solutions having
the same expectation value of energy, or even the same eigenvalue - that
doesn't come easily! So, somehow you have to broach these issues.

Thus I'm reluctant to talk about the issues you're raising now. They're
too fancy for this conversation. I'll just whisper to you the approach
I'm implicitly taking towards this question:

>What, then, is a photon's wave-function?

I'm taking it to be a solution of Maxwell's equations, either described
using the vector potential in some fixed gauge, or perhaps even better
for the present purposes, using the electric and magnetic fields. I bet
people who do quantum optics do something like this when they talk about
the wavefunction of a photon, and I don't think it's so bad, despite the
objections you note.
---------------------------------------------------------------------

> Now get the frying pan ready and prepare your tie or hat for
> eating.
>
> Bon appetit.

I assume that you are the honour person, you know that obligates
the debt of honour to make in similar cases...

Bon appetit.

---
Aleksandr Timofeev

>
> Franz Heymann

c.h.thompson

unread,
May 29, 2001, 9:41:47 AM5/29/01
to

Aleksandr Timofeev <a_n_ti...@my-deja.com> wrote
> "franz heymann" <franz....@care4free.net> wrote

> > > > But you froget that a whole century has passed in the


> > > > mean time and that no prediction made by quantum
> > > > mechanics has ever been proved wrong in any
> > > > experimental test.

Hmm ... But how do you explain Lippmann fringes? I asked a couple of times
and am waiting for an answer.

[skip Franz Heymann's exposition of the orthodox confusion. Baez'
confession below is more interesting.]

> All by you written here is attempt of introduction of
> artificial additional hypothesises for saving an absolutely
> excessive hypothesis of a photon.

Too true, but as Baez explains, these people who believe it exists -- that
the whole mathematical fantasy is worth the effort -- have had their brains
systematically turned to jelly! They are no longer able to comprehend
common sense.

I'm with you, Aleksandr! John Baez' message amounts to a confession of
systematic attempted brainwashing of captive students. It should be
punishable by law!

franz heymann

unread,
May 29, 2001, 12:07:45 PM5/29/01
to

c.h.thompson <c.h.th...@newscientist.net> wrote in message
news:3b13a...@news2.vip.uk.com...
>
[Skip. because I have an isolated query only]

>
> Hmm ... But how do you explain Lippmann fringes? I asked a
couple of times
> and am waiting for an answer.

Since past performance would indicate that you probably do not
know very much about them, I would appreciate it if you were to
explain your understanding of what Lippmann fringes are about, so
that we don't start arguing from different platforms.

[Skip. because I had an isolated query only]

Franz Heymann


c.h.thompson

unread,
May 30, 2001, 3:43:00 AM5/30/01
to

franz heymann <franz....@care4free.net> wrote
>
> c.h.thompson <c.h.th...@newscientist.net> wrote
> >
> > Hmm ... But how do you explain Lippmann fringes?
> > I asked a couple of times and am waiting for an answer.
>
> Since past performance would indicate that you
> probably do not know very much about them,

Why the insulting tone?

> I would appreciate it if you were to
> explain your understanding of what Lippmann fringes
> are about, so that we don't start arguing from different
> platforms.

They are interference patterns due to standing waves set up between light
beams travelling in opposite directions. Thinking about it, they are not so
very remarkable these days, but they were first observed well before we were
able to do tricks such as trapping ions using coherent laser beams.

The only source of information I have on them is Herbert Ives' paper, his
Rumpford Medal Lecture 1951, "Adventures with Standing Light Waves", which
is reproduced in full in Dean Turner and Richard Hazelett, "The Einstein
Myth and the Ives Papers", Devin-Adair 1979.

In 1890 Otto Wiener did an experiment using a beam of light perpendicular to
mirror. A very thin photographic film was placed at a small angle to
surface. The film passes through successive nodes and internodes and an
interference pattern is seen when it is developed.

Later Gabriel Lippmann developed a colour photography method using standing
waves. Ives did his PhD on it and improved understanding. He did many
experiments, using very thick layers of photographic emulsion, and was able
to process them so as to produce laminated solids that reflected the
wavelength that formed them if illuminated perpendicular to the surfaces but
different wavelengths at different angles. A detailed study was later done
using the photoelectric effect to map wave intensity. Ives produced a
complete explanation, using pure wave theory. He did not see any way that
his work could be explained using the photon concept.

In point of fact, I possibly DO know how quantum optics would try and
explain the fringes, but the explanation is, as Hubble would have said,
"forced".

keith stein

unread,
May 30, 2001, 7:29:03 AM5/30/01
to
Thanks for that fascinating answer Caroline,
and glad that that horrid Mr Heymann bullied you into it,
now
and in gratitude, i've posted another question for you ?

a "don't know" would be sufficient answer,
if you're busy eh!

keith stein

"c.h.thompson" <c.h.th...@newscientist.net> wrote in message

news:3b14b...@news1.vip.uk.com...

So that's what "Lippmann fringes" are.

That was very interesting Caroline.
From your description i gather they are caused by
interference of the incident wave with the reflected wave.

I have no difficulty understanding that, because
one always gets an alogous effect with acoustic
waves in gas pipes, and i played with them for years eh!

Caroline are you aware of the important difference between
Maxwell's theory as written by Maxwell and Maxwell's
theory as presented by Einstein in his 1905 SR Paper ?


Aleksandr Timofeev

unread,
May 30, 2001, 8:56:23 AM5/30/01
to
"franz heymann" <franz....@care4free.net> wrote in message news:<3b0ff3b5$0$15026$cc9e...@news.dial.pipex.com>...
[snip]

> Fourthly, a macroscopic radio wave is the wavefunction of an
> extremely large assembly of photons, all coherently sharing the
> same wave function.

This your error concept John Baez already has refuted very
convincingly.

> Detecting one photon at one of the antennae
> simultaneously with the detection of another (coherent) photon at
> another antenna is then possible.

[snip all]

Here you introduce an artificial additional (auxiliary)
hypothesis of simultaneous passing through two slots of two
photons.

----------------------------------------------------
This hypothesis is a weird heresy from a point of view
orthodox physics.
----------------------------------------------------

In all text-books physicists the photon passing
through both slot simultaneously is circumscribed.

Similar scientifically fancy fairy tale about an interference

of photons on two slots is adduced in the book:

Richard Feynman "THE CHARACTER OF PHISICAL LAW";
A series of lectures recorded by the BBC at Cornell University USA;
Cox and Wynman LTD, London, 1965

this fairy tale is refuted by experimental existence of a virtual
interference. ;o)


The purpose of the given article is the proof of an
inaccuracy of representation about the classical
interpretation of a phenomenon of an interference
and explanation principles of virtual interference.

----------------------------------------------------------------
From: Duality Light (X...@MailAndNews.com)
Subject: Debunking Duality Of Light
Newsgroups: sci.physics
Date: 2001-05-07 07:39:52 PST

Foton or Logic error
in the classical interpretation of a phenomenon
of an interference of electromagnetic waves in an interferometer

The gnoseological scheme: the person - natural phenomenon

A. Person. The person and human brain are macroscopic systems.
With the help of of sense organs the brain can analyze the
information recorded on macroscopic structures. The limitations
of sense organs are overcome with the help of of macroscopic
devices.
B. The remote source emits electromagnetic radiation - natural
phenomenon.
C. The interferometer is the device transforming energy of
electromagnetic radiation in a macroscopic image of an
interference picture.

-------------------------------
Let's analyze principles of operation of an interferometer with
the help of of maximum simplified basic gnoseological scheme.

I select the constituents of an interferometer:

1. Screen with two slots (antenna of receivers);
2. Device transforming two flows of electromagnetic radiation from
two slots (antennas of receivers) in a macroscopic image of an
interference picture;
3. Macroscopic image of an interference picture.

Here I shall specify a source (radical) of a logic error of the
interpretation of a phenomenon of an interference on the basis of an
error hypothesis (chimera) of a light photon.

-------------------------------
We shall begin from 3 item:
The macroscopic image of an interference picture can be created
only by

quantum processes of transformations

_ inside _ of quantum microsystems making a mosaic record of a
macroscopic image. The quantum microsystems can exchange (absorb
and emit) energy only by quantum portions. This energy is absorbed and
is emited as electromagnetic waves. Creations of a macroscopic image do
not need a hypothesis of a light photon, but just in this place this
hypothesis occurs ostensibly for explanation of a phenomenon of an
interference, though in her there is no necessity absolutely.
The logic error is done just in this place, the further discuss of
a problem will remove all doupts in that one.

-------------------------------
Let's consider item 1.
Screen with two slots or two antennas of receivers.
Now there are two kinds of interferometers. For understanding
distinctions (differences) between them let's play by terms -
an interference in real time and virtual interference:
a) Everyone know about existence of a phenomenon of an
interference in real time is there is a classical phenomenon
of an interference.
b) Presently there is a new kind of an interference - so-called
postponed in time or virtual interference, i.e. abstractly or
mathematically realizabled interference in the computer.
---------------------
In this place we can and should clearly understand main idea, that
for a phenomenon of an interference the state information of an
electromagnetic field in space of slots of a screen (or on antennas
of receivers) interferometer is important only, all further
processes are causal corollaries of this information.
It is the experimental fact confirmed by existence of a virtual
interference.


---------------------
Here for the first time clearly emerges, that for a hypothetical
particle of a photon there is no necessity to pass simultaneously
through both slots (antennas), since the virtual interference
abstractly or mathematically will be realized in the computer at
any convenient time hereafter. !!! It is the experimental fact!!!

How the admirers of a hypothesis of photons now will explain
an interference?
---------------------

We can simultaneously record the information reflecting a state
of an electromagnetic field in space of each slot (from the
antenna) on a magnetic tape, it is natural that for each slot
(antenna) we use a separate magnetic tape. Then in any time,
convenient for us, we input the information from these macroscopic
magnetic tapes in the computer and mathematically on any required
(demanded) algorithm (which can be changed at any time) we obtain
an interference in representation, necessary for us.
At use of the given method the interference picture represents the
pure abstract information, then this information the macroscopic
computer can transform to the form accessible for the analysis by
a macroscopic system - by the person.

The absence of influence of a state of an electromagnetic field
in space of one slot (antenna) on a state in other one becomes
perfect obvious, since a limit of a distance between slots
(antennas) experimentally is not reached, and this distance can be
made _physically vast_ on a comparison with a wavelength.
This circumstance makes completely inconsistent a hypothesis of
a photon, since the photon should have physically absurd vast sizes
for a simultaneous contact to both slots.
Further, the experimental fact of existence of a virtual
interference basically excludes necessity of simultaneous passing
of a photon through both slots. There is no necessity to pass
through both slots/antennas or one slot/antenna at all!!!

-------------------------------
Now we shall consider item 2.
The device transforming in real time a part of energy of flows
of electromagnetic radiation from two slots (antennas of receivers)
into a macroscopic image of an interference picture - this is a
classical phenomenon of an interference. In a classical optical
interferometer the image of an interference picture can be either
on a photo or on diffusely dispersing a screen or can be project
immediately on a retina of an eye.
The macroscopic image of an interference picture can be created
only by quantum processes of transformation(conversion) _ inside _
of quantum microsystems making a mosaic record of a macroscopic image
at the expense of energy of flows of electromagnetic radiation from
two slots.


The quantum microsystems can absorb energy only by quantum
portions. This energy is absorbed as electromagnetic waves by
quantum microsystems at random coincidence of orientation of a
spatial dynamic configuration of a quantum microsystem with
orientation of an electromagnetic wave. Analogy between a quantum
microsystem and directional antenna here is conducted in an

obvious kind. These random coincidences are improbable, therefore
for obtaining an image are required or enough strong flows of
energy of electromagnetic radiation or large periods for
accumulation of an image.
Briefly, constructions of a macroscopic image need certain
quantity of energy, also it is necessary to take into account and
efficiency of transformation. Creations of a macroscopic image do
not need a hypothesis of a light photon, but just in this place of
explanation of a phenomenon of an interference this hypothesis is
introduced, though in this hypothesis absolutely there is no
necessity. The hypothesis of a light photon theoretically is
excessive, since the virtual interference abstractly or
mathematically is realized in the computer.

The logic error of introduction of a hypothesis of a light
photon in _ classical _ explanation of creation of a macroscopic
image of an interference picture is hidden in error understanding
of the gear of conversion of energy of an electromagnetic wave
during an absorption of this energy by quantum microsystems.
Once again, the quantum microsystems absorb energy of
electromagnetic waves at random coincidence of orientation of a

spatial dynamic configuration of a quantum microsystem with
orientation of an electromagnetic wave.

-------------------------------------------------
Conclusion:

The purpose of the given article was the proof of an
inaccuracy of representation about a duality of physical
properties of light.
Light is wave process always and in all cases.
The nature, ambient us, consists of quantum microsystems,
therefore any phenomenon can be explain from a point of Plank's
view - quantum microsystems can exchange energy only by quantum
portions. This energy is absorbed and is emited only as
electromagnetic waves.

Photoeffect, Raman and Compton effects and all other phenomena
have physically correct explanation only from the point of Plank's
view.
The classical interpretations of a photoeffect and Compton
effect are error.
These interpretations were offered when there was no quantum
mechanics and radio physics. In that time the principles of operation
of transmitting and receiving devices were poorly clear and known
in detailses. The processes in solid bodies and structure of solid
bodies in that time were unintelligible. The quantum theory of a
structure of substance (physical chemistry) was not in that time.

But the pceudo-scientific imaginations as the classical
interpretations of a photoeffect and Compton effect on the
basis of a hypothesis of a photon are alive until now. Why?

------------------------------------------------------------

Charles Francis

unread,
May 30, 2001, 9:18:52 AM5/30/01
to
In article <3b14b...@news1.vip.uk.com>, c.h.thompson <c.h.thompson@new
scientist.net> writes
>

>franz heymann <franz....@care4free.net> wrote
>>
>> c.h.thompson <c.h.th...@newscientist.net> wrote
>> >
>> > Hmm ... But how do you explain Lippmann fringes?

>They are interference patterns due to standing waves set up between light


>beams travelling in opposite directions. Thinking about it, they are not so
>very remarkable these days, but they were first observed well before we were
>able to do tricks such as trapping ions using coherent laser beams.
>
>

>In 1890 Otto Wiener did an experiment using a beam of light perpendicular to
>mirror. A very thin photographic film was placed at a small angle to
>surface. The film passes through successive nodes and internodes and an
>interference pattern is seen when it is developed.

In quantum mechanics, as I gather you know, the wave pattern for a
photon does not interfere with the wave pattern for another photon. But
it can interfere with the wave pattern for itself after reflection. Here
you do not have two distinct beams travelling in opposite directions,
but the same beam reflected back on itself.

>Later Gabriel Lippmann developed a colour photography method using standing
>waves. Ives did his PhD on it and improved understanding. He did many
>experiments, using very thick layers of photographic emulsion, and was able
>to process them so as to produce laminated solids that reflected the
>wavelength that formed them if illuminated perpendicular to the surfaces but
>different wavelengths at different angles. A detailed study was later done
>using the photoelectric effect to map wave intensity. Ives produced a
>complete explanation, using pure wave theory. He did not see any way that
>his work could be explained using the photon concept.

When there are a large number of photons, the photon concept will give
exactly the same experimental result as classical wave theory. Only if
you could arrange the experiment in such a way as to allow the photons
to be releases one at a time, so that you could see a discrete pattern
appear on the photographic emulsion when a few photons have been through
the system, gradually building up towards an apparently continuous
pattern when many photons have been seen.

Of course I have no information on the practicality of such an
experiment as described here, but I am given to understand that
equivalent experiments (picking up one photon at a time) have been done
in other cases, such as Young's slits.


Regards

--
Charles Francis

franz heymann

unread,
May 30, 2001, 6:54:16 AM5/30/01
to

c.h.thompson <c.h.th...@newscientist.net> wrote in message
news:3b14b...@news1.vip.uk.com...

>
> franz heymann <franz....@care4free.net> wrote
> >
> > c.h.thompson <c.h.th...@newscientist.net> wrote
> > >
> > > Hmm ... But how do you explain Lippmann fringes?
> > > I asked a couple of times and am waiting for an answer.
> >
> > Since past performance would indicate that you
> > probably do not know very much about them,
>
> Why the insulting tone?

A reasonable guess, enabled by experience.


>
> > I would appreciate it if you were to
> > explain your understanding of what Lippmann fringes
> > are about, so that we don't start arguing from different
> > platforms.
>
> They are interference patterns due to standing waves set up
between light
> beams travelling in opposite directions. Thinking about it,
they are not so
> very remarkable these days, but they were first observed well
before we were
> able to do tricks such as trapping ions using coherent laser
beams.

They were first observed more than a century ago.
Every laser has a standing wave of light between the mirrors.
There is indeed nothing at all remarkable about standing waves.
A standing wave of photons is
simply a trivial example of a wave function which is an
eigenstate of energy but not an eigenstate of momentum.

>
> The only source of information I have on them is Herbert Ives'
paper, his
> Rumpford Medal Lecture 1951, "Adventures with Standing Light
Waves", which
> is reproduced in full in Dean Turner and Richard Hazelett, "The
Einstein
> Myth and the Ives Papers", Devin-Adair 1979.
>
> In 1890 Otto Wiener did an experiment using a beam of light
perpendicular to
> mirror. A very thin photographic film was placed at a small
>angle to
> surface. The film passes through successive nodes and
internodes and an
> interference pattern is seen when it is developed.

Interesting but as expected.


>
> Later Gabriel Lippmann developed a colour photography method
>using standing
> waves. Ives did his PhD on it and improved understanding. He
>did many
> experiments, using very thick layers of photographic emulsion,
>and was able
> to process them so as to produce laminated solids that
>reflected the
> wavelength that formed them if illuminated perpendicular to the
> surfaces but
> different wavelengths at different angles. A detailed study
> was later done
> using the photoelectric effect to map wave intensity. Ives
> produced a
> complete explanation, using pure wave theory. He did not see
> any way that
> his work could be explained using the photon concept.

What was his problem? I see no problem unless the detector is
placed too far from the mirror.

> In point of fact, I possibly DO know how quantum optics would
try and
> explain the fringes, but the explanation is, as Hubble would
have said,
> "forced".

If indeed you possibly know how to explain the fringes, please
give the explanation, and indicate what is "forced" about the
explanation. If you do this, I will give you a correct
explanation next time round if yours is incorrect. If yours is
correct, I will of course acknowledge it.

None of what you wrote contains anything contrary to what is
expected from QM.

Franz Heymann


Zundark

unread,
May 30, 2001, 12:11:20 PM5/30/01
to
Charles Francis wrote:

> In article <MPG.157b561c9...@news.freeserve.net>, Zundark
> <zund...@alberteinstein.co.uk> writes
> >Charles Francis wrote:
> >
> >> The wave is only used for the calculation of probability. It
> >> is a complex valued function and as such no more represents physical
> >> reality than the square root of minus one. A calculational device,
> >> that is all.
> >
> >Why do you assume that a complex-valued function cannot represent
> >physical reality?

[snip]


> The scientific use of mathematics in physics means that we must have an
> empirical basis for it. We have an empirical basis for integer numbers.
> But we have no empirical basis for complex numbers. If it comes to it we
> do not even have an empirical basis for real numbers, and any claim that
> they exist in nature is as scientific as the theory of phlogiston.

But surely any claim that they *don't* exist in nature is equally
unscientific. We simply don't know.

But even if we accept for the moment that real numbers don't exist
in nature, I don't see how you can dismiss the possibility that the
wavefunction represents physical reality simply because it's a complex-
valued function. How do you know that the complex-valued function
isn't just an approximation to something else, as yet undiscovered?

--
Zundark

franz heymann

unread,
May 30, 2001, 11:07:42 AM5/30/01
to

Aleksandr Timofeev <a_n_ti...@my-deja.com> wrote in message
news:e16a4a22.01053...@posting.google.com...
>
[Snip all]

Good luck to you. If you had said what you wanted to say in a
tenth of the number of words you used, I might have read it.
Life is too short to cope with that amount of verbiage.

Franz Heymann


c.h.thompson

unread,
May 30, 2001, 9:18:42 AM5/30/01
to

keith stein <ks012...@blueyonder.co.uk> wrote

> Thanks for that fascinating answer Caroline,

Thanks for the encouragement!

> and glad that that horrid Mr Heymann bullied
> you into it, now
> and in gratitude, i've posted another question for you ?
>
> a "don't know" would be sufficient answer,
> if you're busy eh!

[skip my explanation of Lippmann fringes, based on a paper by Herbert Ives]

> So that's what "Lippmann fringes" are.
>
> That was very interesting Caroline.
> From your description i gather they are caused by
> interference of the incident wave with the reflected wave.

That's right.

> I have no difficulty understanding that, because
> one always gets an alogous effect with acoustic
> waves in gas pipes, and i played with them for
> years eh!
>
> Caroline are you aware of the important difference
> between Maxwell's theory as written by Maxwell
> and Maxwell's theory as presented by Einstein in
> his 1905 SR Paper ?

Not really. I do know that Maxwell worked in quarternions and did not write
"Maxwell's equations" in the form we know them. Some of his ideas got lost
in the change over.

But basically I'm not trying to build on Maxwell's maths. I'm trying to
understand the intuitive ideas he had, and the experimental facts that led
up to them. Some of the ideas can be deduced from the maths, and this
shows me that he may have been a little confused. He tried to apply theory
that he had confirmed in the lab to what might happen to oscillating wave
systems freely propagating in open space. He was very short on experimental
facts here. His ideas on light, therefore, are merely his opinion, not
necessarily better than yours or mine.

Cheers

Charles Francis

unread,
May 30, 2001, 2:58:46 PM5/30/01
to
In article <3b0ff3b5$0$15026$cc9e...@news.dial.pipex.com>, franz
heymann <franz....@care4free.net> writes

>Fourthly, a macroscopic radio wave is the wavefunction of an
>extremely large assembly of photons, all coherently sharing the
>same wave function. Detecting one photon at one of the antennae
>simultaneously with the detection of another (coherent) photon at
>another antenna is then possible.

To be more precise a macroscopic radio wave is the expectation of a
field operator acting on a large assembly of photons. I am not sure that
such an assembly can meaningfully be described as having a unique wave
function, distinct from the wave functions of the individual photons.
You can have an incoherent assembly, just as you can have incoherent
light, but generally it is not nice to listen to the radio picking up
static.

Regards

--
Charles Francis

c.h.thompson

unread,
May 30, 2001, 3:11:08 PM5/30/01
to

franz heymann <franz....@care4free.net>

>
> Good luck to you. If you had said what you wanted
> to say in a tenth of the number of words you used,
> I might have read it.
> Life is too short to cope with that amount of verbiage.

You, Franz, if you remember, had given us an enlightening description of
long baseline interference, including the passages:

> Fourthly, a macroscopic radio wave is the wavefunction
> of an extremely large assembly of photons, all coherently

> sharing the same wave function. Detecting one photon


> at one of the antennae simultaneously with the detection
> of another (coherent) photon at
> another antenna is then possible.
>

> Fifthly, in the classical limit, the behaviour of such a
> coherent assembly of photons has been proven to
> tend to just the behaviour one would expect from
> an application of Maxwell's equations.

Aleksandr Timofeev had pointed out that:

"All by you written here is attempt of introduction of
artificial additional hypothesises for saving an absolutely
excessive hypothesis of a photon."

which is absolutely true.

He had also quoted a posting from John Baez. It included such illuminating
passages as:

"Education is a process of telling a carefully chosen sequence of lies
in which the amount of deliberate deception gradually tends towards zero.
There is a limit to how much truth someone can absorb all at once without
their brain turning to jelly!"

Couldn't you just admit that waves are what there is -- that the wave
function is totally redundant? Why do you have to carry on attempting to
turn our minds to jelly?

At age 58 I think I'm safely out of the danger zone: I know what I know. It
is the young that you and your friends are damaging.

c.h.thompson

unread,
May 30, 2001, 3:20:51 PM5/30/01
to

franz heymann <franz....@care4free.net> wrote
> c.h.thompson <c.h.th...@newscientist.net> wrote
> > franz heymann <franz....@care4free.net> wrote
> > > c.h.thompson <c.h.th...@newscientist.net> wrote
> > > >
> > > > Hmm ... But how do you explain Lippmann
> > > > fringes? I asked a couple of times and am
> > > > waiting for an answer.
[skip explanation: they are interference patterns between waves travelling
in opposite directions]

> They were first observed more than a century ago.

As indeed I say below ...

> Every laser has a standing wave of light between the mirrors.

I know.

> There is indeed nothing at all remarkable about standing waves.

I agree.

> A standing wave of photons is
> simply a trivial example of a wave function which is an
> eigenstate of energy but not an eigenstate of momentum.

Hmm ... Predictable!

> > The only source of information I have on them is
> > Herbert Ives' paper, his Rumpford Medal Lecture
> > 1951, "Adventures with Standing Light Waves",
> > which is reproduced in full in Dean Turner and
> > Richard Hazelett, "The Einstein Myth and the
> > Ives Papers", Devin-Adair 1979.
> >
> > In 1890 Otto Wiener did an experiment using a
> > beam of light perpendicular to mirror. A very thin

> > photographic film was placed at a smallangle to


> > surface. The film passes through successive nodes
> > and internodes and an interference pattern is seen
> > when it is developed.
>
> Interesting but as expected.

Agreed.

[skip a few words on Lippmann and Ives's own contributions]

> > In point of fact, I possibly DO know how quantum
> > optics would try and explain the fringes, but the
> > explanation is, as Hubble would have said,
> > "forced".
>
> If indeed you possibly know how to explain the fringes,
> please give the explanation, and indicate what is "forced"
> about the explanation. If you do this, I will give you a
> correct explanation next time round if yours is incorrect.
> If yours is correct, I will of course acknowledge it.

No way! See above. I asked YOU for an explanation, and have been waiting a
few days! Since there is a perfectly adequate classical wave explanation
available (as I'm sure you well know!) why should I waste time putting my
neck on the block for your amusement?

Charles Francis

unread,
May 30, 2001, 3:08:26 PM5/30/01
to
In article <MPG.157f3a10a...@news.freeserve.net>, Zundark

<zund...@alberteinstein.co.uk> writes
>Charles Francis wrote:
>
>> In article <MPG.157b561c9...@news.freeserve.net>, Zundark
>> <zund...@alberteinstein.co.uk> writes
>> >Charles Francis wrote:
>> >
>> >> The wave is only used for the calculation of probability. It
>> >> is a complex valued function and as such no more represents physical
>> >> reality than the square root of minus one. A calculational device,
>> >> that is all.
>> >
>> >Why do you assume that a complex-valued function cannot represent
>> >physical reality?
>[snip]
>> The scientific use of mathematics in physics means that we must have an
>> empirical basis for it. We have an empirical basis for integer numbers.
>> But we have no empirical basis for complex numbers. If it comes to it we
>> do not even have an empirical basis for real numbers, and any claim that
>> they exist in nature is as scientific as the theory of phlogiston.
>
>But surely any claim that they *don't* exist in nature is equally
>unscientific. We simply don't know.

To an empiricist science is based on what we can observe. If we cannot
observe it, it is unscientific, and speculation about it either way is
unscientific. I do not need to claim that it is impossible for nature to
have all sorts of bizarre, or even seemingly reasonable properties. I
merely ignore such possibilities, and do not include them in what I
regard as science.

>But even if we accept for the moment that real numbers don't exist
>in nature, I don't see how you can dismiss the possibility that the
>wavefunction represents physical reality simply because it's a complex-
>valued function. How do you know that the complex-valued function
>isn't just an approximation to something else, as yet undiscovered?

I do know that I can "reverse engineer" a complex valued wave function
from an interpretation of quantum mechanics in terms of probabilities,
and so I feel we already know that the wave function is just a
calculational device, designed for the purpose. There is nothing strange
about this. Even when I was at school and I was given a problem in
mechanics, I would write down an equation describing the system, and I
would derive an answer which was undoubtedly true for the system, but
the intermediate lines of the derivation would be valid mathematics, and
yet they could not be translated into English describing anything
physical about the system.

Regards

--
Charles Francis

keith stein

unread,
May 30, 2001, 2:56:46 PM5/30/01
to

"c.h.thompson" <c.h.th...@newscientist.net> wrote in message
news:3b153...@news2.vip.uk.com...

If you take my advice Caroline you get a copy of Maxwell's
"Treatise on Electricity and Magnetism". His vector notation
is a bit dated, but you would find that you could follow it.

Even if i'm wrong about that you would certainly understand
that Maxwell insisted a medium was necessary, and he wrote
NO equations for vacuum.

Whereas Einstein's in his 1905 SR Paper, in describing Maxwell's
theory ASSUMES a medium is not necessary, and includes ONLY
equations for vacuum.

The equations are 'nonsense' without a 'medium', as Maxwell realized.
What is the speed of light relative to without the medium, "nothing"!
The speed of "anything" relative to "nothing" is nonsense,of course!
Maxwell's theory gives the speed relative to the 'medium', and that's
why from first to last Maxwell insisted on the necessity of a 'medium':-

"In several parts of this treatise an attempt has been made to
explain electromagnetic phenomena by means of mechanical action
transmitted from one body to another my means of a medium occupying the
space between them. The undulatory theory of light also assumes the
existence of a medium. We have now to shew that the properties of the
electromagnetic medium are identical with those of the luminiferous
medium.
......if the study of two different branches of science has
independently suggested the idea of a medium .... the evidence for the
physical existence of the medium will be considerable strengthened.
...... and the combination of the optical with the electrical evidence
will produce a conviction of the reality of the medium ...
According to the theory of undulation, there is a material
medium which fills the space between the two bodies, and it is by the
action of contiguous parts of this medium that the energy is passed
on... We must therefore regard the medium as having a finite density."

Clearly Maxwell knew that light travels across interstellar
space, so i can only think that he must have correctly guessed that
there was some finite density gas out there, or was the Hydrogen in
space actually discovered much earlier than i thought? Certainly the
examples Maxwell gives of what he meant by a medium are real substances
e.g. Air, or paraffin, and he doesn't mention 'ether' anywhere.

Maxwell commences his mathematical treatment with this clear definition
of his frame of reference for the transmission of his electomagnetic
waves:-
"Let us next determine the conditions of the propagation of an
electromagnetic disturbance through a uniform medium, which we shall
suppose to be at rest, that is, to have no motion except that which may
be involved in electromagnetic disturbances."

and Maxwell concludes:-
"In fact,whenever energy is transmitted from one body to another
in time, there must be a medium or substance in which the energy exists
after it leasves one body and before it reaches the other,for energy,as
Torricelli remarked,'is a quintessence of so subtile a nature that it
cannot be contained in any vessel except the inmost substance of
material things.' Hence all these theories lead to the conception of a
medium in which the propagation takes place..."Maxwell, 1881

Charles Francis

unread,
May 30, 2001, 3:53:23 PM5/30/01
to
In article <3b154...@news2.vip.uk.com>, c.h.thompson <c.h.thompson@new
scientist.net> writes

>He had also quoted a posting from John Baez. It included such illuminating
>passages as:
>
>"Education is a process of telling a carefully chosen sequence of lies
>in which the amount of deliberate deception gradually tends towards zero.
>There is a limit to how much truth someone can absorb all at once without
>their brain turning to jelly!"
>
>Couldn't you just admit that waves are what there is -- that the wave
>function is totally redundant? Why do you have to carry on attempting to
>turn our minds to jelly?
>
>At age 58 I think I'm safely out of the danger zone: I know what I know. It
>is the young that you and your friends are damaging.

You do not know what you think you know. You just believe a set of lies
you were told when you were younger, for fear your brain turns to jelly
in attempting to understand quantum mechanics. But really qm is just
statistics without a random variable. It sounds like a weird thing to
try if you are used to having a random variable, and it comes out with
some weird results, like interference patterns. But be afraid of it.
There is no reason why nature should have described itself with numbers
in the first place. We describe it with numbers.
Regards

--
Charles Francis

franz heymann

unread,
May 30, 2001, 4:30:33 PM5/30/01
to

c.h.thompson <c.h.th...@newscientist.net> wrote in message
news:3b1547e5$1...@news2.vip.uk.com...

And the words used classical explanation are also the words used
in the quantum mechanical explanation, on the assumption that the
distance between the mirror and the detecting film is small
compared to the coherence length of the photon. The coherence
length of a photon from a typical dipole atomic transition may be
in the region of a few hundred cm., so there is quite a latitude
available for the positioning of the film.

Franz Heymann


franz heymann

unread,
May 30, 2001, 4:42:05 PM5/30/01
to

c.h.thompson <c.h.th...@newscientist.net> wrote in message
news:3b154...@news2.vip.uk.com...
>
[Snip all except the remark relevant to the statement I wish to
make]

>
> At age 58 I think I'm safely out of the danger zone: I know
what I know. It
> is the young that you and your friends are damaging.

I am 76 and I am still learning daily. If you have isolated
yourself from broadening your understanding of physics, at 58,
then I have nothing more to say in this thread.

Goodbye

Franz Heymann

franz heymann

unread,
May 30, 2001, 4:51:49 PM5/30/01
to

franz heymann <franz....@care4free.net> wrote in message
news:3b155c34$0$12250$cc9e...@news.dial.pipex.com...

I don't know why this message ended up in a new thread.

In conformity with my remarks in another thread that your mind
has, as you yourself confessed, become ossified, I won't be
replying to any remarks you might make in response to my last
posting here.

Goodbye,

Franz Heymann


Richard Herring

unread,
May 31, 2001, 6:27:19 AM5/31/01
to
In article <3b153...@news2.vip.uk.com>, c.h.thompson (c.h.th...@newscientist.net) wrote:

> keith stein <ks012...@blueyonder.co.uk> wrote

> > Caroline are you aware of the important difference
> > between Maxwell's theory as written by Maxwell
> > and Maxwell's theory as presented by Einstein in
> > his 1905 SR Paper ?

> Not really. I do know that Maxwell worked in quarternions and did not write
> "Maxwell's equations" in the form we know them. Some of his ideas got lost
> in the change over.

That old canard again. *What* got lost in the change over?
And what, if any, are the empirically measurable consequences?

> But basically I'm not trying to build on Maxwell's maths. I'm trying to
> understand the intuitive ideas he had, and the experimental facts that led
> up to them.

In that case, you might do better to study Faraday. He was the one
who derived most of the intuitive ideas from the experimental facts,
with little mathematics and great success. Maxwell's role was to
give the ideas a mathematical skeleton. Having done so, he noticed
a mathematical inconsistency that Faraday had not discovered
experimentally. This led him to postulate the existence of a
displacement current, without which we would have no EM waves.

> Some of the ideas can be deduced from the maths, and this
> shows me that he may have been a little confused. He tried to apply theory
> that he had confirmed in the lab to what might happen to oscillating wave
> systems freely propagating in open space. He was very short on experimental
> facts here. His ideas on light, therefore, are merely his opinion, not
> necessarily better than yours or mine.

His ideas are indeed not necessarily the final story. But there
have been plenty of scientists since Faraday and Maxwell, who
have extended the domain of experimental observations well beyond
the Royal Institution laboratories.

And his equations-in-the-form-we-know-them make quantitative and
falsifiable predictions which agree with those observations. That is
empirical fact, better than anyone's mere opinion.

--
Richard Herring | <richard...@baesystems.com>

Aleksandr Timofeev

unread,
May 31, 2001, 10:53:04 AM5/31/01
to
"franz heymann" <franz....@care4free.net> wrote in message news:<3b152d4b$0$15030$cc9e...@news.dial.pipex.com>...
Well, we shall continue

The photon is mathematical abstraction, which in the implicit
(latent) form reflects existence of discrete power levels in
microsystems and as a corollary a capability of exchange by
electromagnetic energy between systems only by discrete portions.

You never can provick, that the microsystem swallows or radiates
electromagnetic energy as particles of photons. This problem is
similar to a problem of a proof of existence of a God. For this reason
it is a problem of a Religious faith. But the fruitfulness of your
life depends on selection of a religious physical faith. If you will
select as a religious physical faith a physical chimera, whether you
reach outcomes, scheduled by you, in your life?


The Nobel Laureate, Irving Langmuir, coined the term "pathological
science" for "the science of things that aren't so". :-(

Einstein warned: "Most mistakes in philosophy and logic occur because
the human mind is apt to take the symbol for reality". ;o)


When I had your age, I was not skilful independently to think. My
consciousness was overloaded another's physical and mathematical ideas.
These ideas seem to me the best reachings of human spirit, limit of
perfection. The existing theories were represented me necessary and
sufficient means for the solution of all possible problems.

Since student's years I was occupied in searches of the "honour"
solution for the n-body problem. Uncountable unsuccessful attempts for
the solution of the n-body problem have convinced me of boundedness of
capabilities of the existing orthodox theories. Life do not stand on
a place, with the cource of time I was learned independently to think,
I have become to overcome _stereotypes _physical and _mathematical
_thinking.

I have come to the conclusion, that the power and the quality of the
physical theory is determined by its ability to the "honour" solution
for the n-body problem. The modern orthodox physical school persistently
ignores this problem, since it cannot give the solution because of
boundedness of the existing theories. The ignoring of the problem of
the "honour" solution for n-body problem is ignoring fundamental
problems of an inconsistency and antipathy of modern physics.

From point of view of the "honour" solution for the n-body problem:

1. The best rating has the thermodynamics.

2. The Newton's theory allows to solve a three - body problem.

3. GR has not the "honour" solution for the two-body problem!!!

4. In regularities found by me, is shown one from the possible
approaches to the "honour" solution for the n-body problem. This
approach does not lean in the explicit form on the modern physical
theories of a gravitation. But this approach leans on radiophysical
methods for non-linear systems.

Here has a place a natural physical paradox for which does
not exist explanations until now.

Here has a place of a happy capability for genuine and lucky
physicists to try the forces in explanation of this paradox.
I think a prize for good luck will be very high.

If you have good physical intuition, you can understand that the
adduced below text is the fatal sentence to all official theories of
a gravitation. Official does not mean true.


. THE SYMMETRY INSIDE THE SOLAR SYSTEM

GRAVITATION. THE EXPERIMENTAL FACTS AND PREDICTIONS

Abstract. The empirical law connecting values of planetary masses in the
Solar system is demonstrated and is analyzed. A characteristic property
of this law is the existence of groups consisting from four planets. The
law allows to predict existence and properties of three unknown planets
inside the Solar system. This law can serve the useful tool for a research
of extra-solar planetary systems.

1. Empirical gravitational regularities of a symmetry in the Solar System

1.1. Magic ratios of linear combinations of planetary masses

Here are the most reliable values of the Solar System [1] planetary
masses that can be experimentally obtained by celestial mechanics:

Table I
Planetary masses and Ratios of linear combinations of masses

Planet Symbol Mass | Ratio Exact Rounded
used for value | considered value ratio
each planet Earth=1 | of the ratio
. |
Jupiter MJU or 1 317.735 |(MJU+MSA)/(MUR+MNE) = 12.9959 ~ 13
Saturn MSA or 2 95.147 | MJU/(MUR+MNE) = 10.0010 ~ 10
Neptune MNE or 3 17.23 | MSA/(MUR+MNE) = 2.9948 ~ 3
Uranus MUR or 4 14.54 | (MJU+MSA)/MNE = 23.9630 ~ 24
Earth MTE or 5 1.000 | MUR/(MTE+MVE) = 8.0110 ~ 8
Venus MVE or 6 0.815 | (MNE+MUR)/MVE = 38.9816 ~ 39
Mars MMA or 7 0.108 | (MTE+MVE)/MME = 33.0000 ~ 33
Mercury MME or 8 0.055 | MVE/(MMA+MME) = 5.0000 ~ 5

The difference between computed values of ratios and the closest
integer can possibly be explained by an effect similar (Francis Aston 1920)
to mass modification caused by dense packing in atom nucleii. The planetary
masses are measured with some errors also.

1.2. Chiral symmetry ratios of linear combinations of the planetary masses

When organised graphically, the ratios [2] of linear combinations of
the planetary masses considered, reveal a chain of gravitational
correlations between triples of planets possessing chiral symmetry:

Table II
Chiral symmetry ratios of linear combinations of the planetary masses

10
I<----------->|
I 13 |
I<==============>I
I | I
? 39 I | I
|<----------------->I 33 |<---------------->I 24 | I
| |<------------------>I |<----------------->I
| | I ? | | I 5 | | I 8 | | I 3 | | I
| | I<====>| | I<====>| | I<====>| | I<====>| | I
| | I | | I | | I | | I | | I
10 9 I 8 7 I 6 5 I 4 3 I 2 1 I
I | | I | | I | | I | | I
I Mercury MarsI Venus EarthI Uran NepI Saturn JupiterI
I I I I I
10+9 8+7 6+5 4+3 2+1
ln(mass)
- - -------------------------------------------------------------->

The following symbols here are used in this graphic:

MSA + MJU <-> 2 + 1; MUR + MNE <-> 4 + 3;
MVE + MTE <-> 6 + 5; MME + MMA <-> 8 + 7;
MJU <-> 1; MSA <-> 2; MNE <-> 3; MUR <-> 4;
MTE <-> 5; MVE <-> 6; MMA <-> 7; MME <-> 8;

5
Direct gravitational correlation - <====>;
33
Reverse gravitational correlation - <---------->


Note: Here it is necessary to understand exclusive importance of
the numbers Fibonacci for gravitational regularities inside
the Solar system in common case:

If you look at direct gravitational connections than you will see the
following numbers: 3, 5, 8, 13.
For the third hypothetical quad there should be now following numbers
accordingly: 21 and 34.

1.3. Formula for pairs of conjugate gravitational correlations.

We shall name "pairs of conjugate gravitational correlations" the
following pairs of values that can be identified on the previous graph:

33,5 39,8 24,3 10,13

We shall now consider relating of sums of those pairs of conjugate
gravitational correlations with squares of natural numbers:

33+5=6^2+2 39+8=7^2-2 24+3=5^2+2 10+13=5^2-2

+2 -2 +2 -2

From these relations, a common formula for the sums of the pairs
of conjugate direct and reverse gravitational correlations can be
established:

(value of reverse correlation)+(value of direct correlation)=n^2 +/- 2

To some extent, this formula is analog to Balmer's formula for
spectral series of the Hydrogen atom. The analysis of the chained series
of conjugate gravitational correlations clearly reveals here a periodic
alternance of the sign before number 2.

1.4. Gravitational correlations for groups of four planets.

For a long time astronomers have been aware of dynamic relations
in celestial bodies in groups of four, in the stable gravitational
system which the Solar System presents us with. On this specific
criterion and on some other dynamic criterions stemming from celestial
mechanics, we can select two groups of four planets in the Solar System.
The planets of the Terrestrial group are: Earth, Venus, Mars and
Mercury. The planets of the Jovian group are: Jupiter, Saturn, Neptune
and Uranus. The empirical facts discovered here indirectly confirm the
existence of further relations.

For the group of planets Earth, Venus, Mars and Mercury
((n^2 + 2);(n ^ 2 - 2)) the relationship is established in the
following manner:

( 33 + 5) + (39 + 8) = 6 ^ 2 + 7 ^ 2 = 9 ^ 2 + 2 ^ 2 = 85

For the group of planets Jupiter, Saturn, Neptune and Uranus
((n ^ 2 + 2); (n ^ 2 - 2)) the relationship is established in the
following manner:

( 10 + 13) + (24 + 3) = 5 ^ 2 + 5 ^ 2 = 7 ^ 2 + 1 ^ 2 = 50

In each of the groups considered, there is a higher pair
(n ^ 2-2) and lower pair of planets (m ^ 2 + 2). Therefore, a
possibility seems to exist to derivate various combinations of these
pairs to obtain mixed combinations from these two groups of four
planets. In our particular case, only the combination of the two lower
pairs ((n ^ 2 + 2); (m ^ 2 + 2)) Neptune, Uranus, Mars and Mercury,
forming a mixed group, allows a correlation to be determined:

( 33 + 5) + (24 + 3) = 7 ^ 2 + 4 ^ 2 = 8 ^ 2 + 1 ^ 2 = 65

Some conclusions:

The considered relations can be expressed as the following formula:

(sum values of all correlations of the given group) = k^2+l^2=m^2+n^2

What is remarkable in these correlations by groups of four planets, is
that the sum of the pairs of conjugate gravitationnal correlations are
equal in each case to natural numbers (50, 65, 85) which are the first
terms of a sequence of natural numbers, which are the sum of two pairs
of squares of natural numbers. Please look Diophantus's theorem of a
number theory (III, 19). Here is the beginning of this series:

! ! !
number 1 25 50 65 85 100 125 130 145 169 170 185 200 205 221 225 250 260

1 1 5 7 8 9 10 11 11 12 13 13 13 14 14 14 15 15 16
pair 0 0 1 1 2 0 2 3 1 0 1 4 2 3 5 0 5 2

2 0 4 5 7 7 8 10 9 9 12 11 11 10 13 11 12 13 14
pair 1 3 5 4 6 6 5 7 8 5 7 8 10 6 10 9 9 8
---------------------------------------------------------

Dear Franz Heymann, I am ready to discuss of your constructive criticism.

---
Aleksandr Timofeev
http://www.friends-partners.org/~russeds/unknown/astrochem/

> Franz Heymann

Aleksandr Timofeev

unread,
May 31, 2001, 11:17:18 AM5/31/01
to
"franz heymann" <franz....@care4free.net> wrote in message news:<3b152d4b$0$15030$cc9e...@news.dial.pipex.com>...

I leave to a vacation. :o)

Well, we shall continue

--------------------------------------------------------------
From: Common Sense (donot...@interbulletin.bogus)
Subject: Re: Debunking Duality Of Light
Newsgroups: sci.physics
Date: 2001-05-10 06:21:08 PST

[snip]
>What? The fact that you are completely wrong escaped your attention? Read a
>beginning physics textbook. Photon phenomena are part of physics,
>wavefunction collapses are also.
>Learn to live with it, because nature keeps providing evidence
>for photons.

I'm just trying to persuade you to face the truth!

[snip]

"I'm afraid the whole scientifically-literate community are little
better than obsesses cranks when it comes to discussing "photons"."

I put here the part of old Deja's article:

This article debunking the classic interpretation
of the photoelectric effect.

---------------------------------------------------
> >> A photon is not point like,
> >
> > I agree with this statement.
> >
> >> it is spread out like a wave. It has a finite wavelength, and it's
> >> momentum is distributed along that length.
> >
> > Excuse me. I doubt hardly of this statement. Why the photon should have a
> >wavelength?
>
> Because it is an electromagnetic wave.
>
> What physical experiments undoubtedly confirm your point of view?
>
> Spectrometers measure wavelength. Interference patterns, and holograms
> depend on wavelength.
>
> >You refer to a phenomenon of a photoeffect. Indicate to me precisely place in
> >the interpretation of a phenomenon of a photoeffect, where clearly it is
> >visible, that the photon has a wavelength.
>
> The photoelectric effect is usually interpreted such that a particle
> collides with the surface of a metal, with enough energy to excite electrons
> from their bound atomic state, into a conduction current. The energy
> required is proportional to the frequency of the photon. You don't have
> frequency of a wave without a wavelength.

In a photoeffect of an appearances happening on the boundary
vacuum - metal, are considered at a falling of light on a surface
of metal:

First, we have light spreaded in vacuo in direction to a surface
of metal.
Second, we have a flat surface of metal.
Third, we have electrons taking off (emitted) in vacuum from a
surface of metal, if the wavelength of incident light is less than
some boundary (critical) wavelength of light.

There are three logically possible paths (if we shall eliminate
vacuum) for theoretical explanation of existence of a critical wavelength
of light:
1. Light is spreaded in space as particles. These particles have a
particle - wave properties and just they are responsible for a photoeffect.
2. Light is spreaded in space as EM waves. Vacuum - metal the
boundary has frequency - selective properties and just this transition
is responsible for a photoeffect.
3. Simultaneously light and Vacuum - metal the transition are
responsible for a photoeffect.

First logic path an explaining phenomenon of a critical wavelength.
======================================= =====================
The equation for a photoeffect was published by J. J. Thomson in
Sallivan's the lectures for the first time in 1903. You consider classical
explanation of a photoeffect, which was given Milena Marich (wife) and A.
Einstein in 1905. From my point of view this explanation physically is
erroneous. In 1905 even the structure of atom was not known. In that time was
indifferent to what of two plants (light or rigid body) to assign quantum
properties.

Second logic path an explaining phenomenon of a critical wavelength.
======================================= =====================
Let's consider the supposition that this phenomenon is explained by
properties of substance which illuminates light.

For the beginning we shall consider explanation on " on fingers ". Any
receiver consists of an antenna and resonator.

Mental experiment 1. An antenna.

We shall show, that the surface stratum of metal represents a good
antenna for reception of electromagnetic waves, which length exceeds in ~ 10
^ 4 times a phase of a crystalline lattice.

A) Let light illuminates a flat screen with a round orifice, which has
a diameter D. What lengths of light waves we can observe behind of a screen? -
Behind Of a screen we can observe waves of light, which length does not exceed
D. Therefore screen is a bandpass filter, if it has an orifice. This filter
effectively passes light with lengths of waves from 0 up to equal D (boundary
maxima of value for a light wave).
Would be erroneous to think, that the waves have length ~ 10 ^ 4*D
cannot be observed behind of a screen. Near to an orifice longer (on a
comparison with D) the light waves damp under the exponential law. These waves
have noticeable amplitude behind of a screen.

B) Let light illuminates a flat grid (have the incorrect form of an
orifice) or lattice. What lengths of light waves we can observe behind of a
flat grid? - Behind Of a flat grid we can observe light waves without
damping, which length does not exceed D. Where D is equal to a maximum
distance(span) between edges of an orifice of a grid. The grid is a bandpass
filter. This filter passes without damping light with lengths of waves from 0
up to equal D (boundary maxima of value for a light wave). Remarkable
advantage of a lattice used as a filter, in a comparison with a unique
orifice, is the essential prize in value of a power stream. Exceeding
boundary value D, the longer light waves (~ 10 ^ 4*D) damp under the
exponential law in direction a perpendicular surface of a lattice. These
waves have noticeable amplitude behind of a lattice.

C) What the flat surface of metal for electromagnetic waves
represents? We can represent a flat surface of metal as a lattice. The
characteristic size of a mesh of this lattice is approximately equal D =
2*10^-10 cm. This lattice is a bandpass filter also. This filter passes
without damping light with lengths of waves from 0 up to equal D (boundary
maxima of value for a light wave).
Exceeding boundary value D, the
longer electromagnetic waves (penetrating through a geometric surface of
metal) damp under the exponential law in direction a perpendicular surface of
a lattice. These waves have noticeable amplitude inside a surface stratum of
metal. Thus for electromagnetic waves of a light range (~ 2*10 ^ -10 * 10 ^
4) surface stratums of metal are represented by some analog of a
multi-element antenna.

Mental experiment 2. Quantum multimode resonator.

Let's consider properties of a rigid body (substance) which
illuminates light. Our crystal is a quantum system. In our crystal many
millions atoms are connected together, and as a result the huge number of
power levels hardly remote from each other inside power zone is received. The
power zones are divided by a significant distance on a comparison with a
distance between power levels inside power zone. The boundary of last power
zone (or last completed level), in which the electrons are, is separated by a
significant distance from power zone of ionization of crystal. The breadth of
a forbidden region is equal to a minimum energy necessary for an exit of an
electron outside of the boundary of crystal. The expansion (wave)
cumulative distribution function circumscribing an electron of an exterior
envelope of atom, exceeds a distance between atoms. When the atoms are
connected one another, some exterior electrons should be shared all atoms of
crystal. For this reason we have given a lot of attentions to exposition of
disrtibuted "antenna" of our "receiver". For electromagnetic waves of a light
range (~ 2*10^-10 * 10^4) surface stratums of metal are represented by some
analog of a multi-element antenna.

Mental experiment 3. Our "receiver" consists of an antenna and quantum
multimode resonator.

Our "receiver" is the quantum multiresonance machine, which has a
property of a reversibility. In the full correspondence by Plank's principle,
our "receiver" accepts and radiates an electromagnetic energy by quantum
portions, which value is determined by an extremely interior device of
"receiver". I repeat once again, this property is a corollary of an interior
device of the given machine and does not require for the explanation of any
additional hypothesises. The property of a reversibility of this machine
allows to minimize an amount of used principles of physical exposition of a
considered phenomenon. We do not have necessity to assign quantum properties
to electromagnetic radiation. For this reason minimum value of frequency in
the equation J. J. Thomson corresponds to fixed own resonance frequency of
ionization of our "receiver". We set up frequency of a radiation under our
"receiver" to receive a photoeffect.

Now there is very great many of diverse quantum electronic devices. For
explanation of principles of their work in electrical circuits there is no
necessity to apply a term a quantized radiation, for these purposes there are
enough of terms of the theory of an electrodynamics.

> Also, the alternate interpretation
> is that there is no particle, only the wave who has its energy stored
> between opposite amplitudes of the wave. Just like a parallel plate
> capacitor, the wavelength separates the opposite electric fields. If the
> differential from +ve ot -ve is great enough, in Volts/meter, then the
> electron in the metal will be excited into the current.
[snip]
The detailses of mechanisms of a radiation and absorption of an
electromagnetic field are not known to us. On my sight the photon is simply
other title for mechanisms of a radiation and absorption of an
electromagnetic field. For this reason the PHOTONS DO NOT EXIST IN A NATURE
or, if it is leasant more to you, in a medium of an electromagnetic field the
photons will be generated as virtual particles.

Corrections welcome.
--------------------------------------------------------------------

> Franz Heymann

c.h.thompson

unread,
Jun 1, 2001, 5:31:43 AM6/1/01
to

keith stein <ks012...@blueyonder.co.uk>
> "c.h.thompson" <c.h.th...@newscientist.net> wrote
> >
> > > [Keith]

> > > So that's what "Lippmann fringes" are.
> > >
> > > That was very interesting Caroline.
> > > From your description i gather they are caused by
> > > interference of the incident wave with the reflected
> > > wave.
> >
> > That's right.
[skip]

> > > Caroline are you aware of the important difference
> > > between Maxwell's theory as written by Maxwell
> > > and Maxwell's theory as presented by Einstein in
> > > his 1905 SR Paper ?
> >
> > Not really. I do know that Maxwell worked in
> > quarternions and did not write "Maxwell's
> > equations" in the form we know them. Some
> > of his ideas got lost in the change over.
>
> If you take my advice Caroline you get a copy of
> Maxwell's "Treatise on Electricity and Magnetism".
[skip]
> ...Maxwell insisted a medium was necessary, and he wrote
> NO equations for vacuum.

Yes, this is what I would have assumed, but don't you think that we now have
a totally different perspective on the world, and this ought to enable us to
construct a very much better model than Maxwell's? I've read some of his
original writing (I think it was in Whittaker's Theories of the Aether).
Basically his aether had to be solid, but I don't think he would have come
up with the same complicated mechanical ideas if he had lived today.

Today we are surrounded by radio waves, microwaves, complicated devices
dependent on sensing electromagnetic signals. We are constantly having to
battle to extract "signal" from "noise", and noise is frequently the
dominant factor. My idea of how Nature works is that the whole thing
depends on "natural" extraction of signal from noise. The quantum theory
idea of "vacuum fluctuations" or the roughly equivalent Stochastic
Electrodynamical one of the "Zero Point Field" are on the right lines.

Maxwell could have had no conception of any of this! Didn't he die before
Herz had even produce radio waves? Before Michelson and Morley had done the
experiment that he had thought would be for ever beyond our reach, searching
for signs of aether drift? He most certainly died before Dayton Miller came
along and (in 1925-6) produced evidence that there WAS and aether. Miller's
aether was not the kind Maxwell was thinking of.

> Whereas Einstein's in his 1905 SR Paper, in describing Maxwell's
> theory ASSUMES a medium is not necessary, and includes ONLY
> equations for vacuum.
>
> The equations are 'nonsense' without a 'medium', as Maxwell realized.
> What is the speed of light relative to without the medium, "nothing"!

[skip]


>
> and Maxwell concludes:-
> "In fact,whenever energy is transmitted from one body to another
> in time, there must be a medium or substance in which the energy exists
> after it leasves one body and before it reaches the other,for energy,as
> Torricelli remarked,'is a quintessence of so subtile a nature that it
> cannot be contained in any vessel except the inmost substance of
> material things.' Hence all these theories lead to the conception of a
> medium in which the propagation takes place..."Maxwell, 1881

Interesting (including the bits I've skipped)!

Note change of address:

Caroline
--
c.h.th...@pgen.net
http://www.aber.ac.uk/~cat


Charles Francis

unread,
Jun 1, 2001, 7:13:17 AM6/1/01
to
In article <3b176...@news2.vip.uk.com>, c.h.thompson
<c.h.th...@pgen.net> writes

>
>keith stein <ks012...@blueyonder.co.uk>
>> "c.h.thompson" <c.h.th...@newscientist.net> wrote
>> >
>> > > [Keith]
>> > > So that's what "Lippmann fringes" are.
>> > >
>> > > That was very interesting Caroline.
>> > > From your description i gather they are caused by
>> > > interference of the incident wave with the reflected
>> > > wave.
>> >
>> > That's right.
>[skip]
>> > > Caroline are you aware of the important difference
>> > > between Maxwell's theory as written by Maxwell
>> > > and Maxwell's theory as presented by Einstein in
>> > > his 1905 SR Paper ?
>> >
>> > Not really. I do know that Maxwell worked in
>> > quarternions and did not write "Maxwell's
>> > equations" in the form we know them. Some
>> > of his ideas got lost in the change over.
>>
>> If you take my advice Caroline you get a copy of
>> Maxwell's "Treatise on Electricity and Magnetism".
>[skip]
>> ...Maxwell insisted a medium was necessary, and he wrote
>> NO equations for vacuum.

Actually Maxwells equation's apply easily to the vacuum. You just put
charge=0. That is how they show e.m. radiation propagating through the
vacuum. Of course if you want an absolute vacuum you can put e.m.
radiation =0 too. Then Maxwell's equations say of the vacuum 0=0, which
is true enough.


>
>Yes, this is what I would have assumed, but don't you think that we now have
>a totally different perspective on the world, and this ought to enable us to
>construct a very much better model than Maxwell's?

We have. It's called quantum electrodynamics and enables us to derive
Maxwell's equations without relying on a mysterious and unexplained
mechanical ether, with properties we now know to be false.

> I've read some of his
>original writing (I think it was in Whittaker's Theories of the Aether).
>Basically his aether had to be solid, but I don't think he would have come
>up with the same complicated mechanical ideas if he had lived today.

Maxwell certainly would have understood qed.

Regards

--
Charles Francis

jmfb...@aol.com

unread,
Jun 1, 2001, 4:48:07 AM6/1/01
to
In article <n3F6Z+AN...@clef.demon.co.uk>,

Charles Francis <cha...@clef.demon.co.uk> wrote:
>In article <3b176...@news2.vip.uk.com>, c.h.thompson
><c.h.th...@pgen.net> writes
<snip>

>> I've read some of his
>>original writing (I think it was in Whittaker's Theories of the Aether).
>>Basically his aether had to be solid, but I don't
>>think he would have come
>>up with the same complicated mechanical ideas if he had lived today.
>
>Maxwell certainly would have understood qed.

You're missing a key assumption of Caroline's. If it wasn't
known back then, it can't be a good theory today. She has
no idea how knowledge is produced.

/BAH


Subtract a hundred and four for e-mail.

G=EMC^2 Glazier

unread,
Jun 1, 2001, 5:28:55 PM6/1/01
to
To All We could be saying one photon and they maybe running around as
coupled pairs. The two slit and a sharp edge proves my point.
There is a theory (in the books) that each photon travels with over a
million "shadow photons" So it you made a million small hole the photon
would have a million patterns on the screen. When 6 trillion photons can
sit on the head of a pin each second,you could say"Who cares about two
or a million best regards to all herb

G=EMC^2 Glazier

unread,
Jun 2, 2001, 9:54:25 AM6/2/01
to
To add to my above post.Einstein liked to equal things.He has the high
energy photon on equal terms as a particle.He has no problem thinking in
the wave-particle direction.He got the Nobel for this thinking. Best
regards to all herb

Matthew Leifer

unread,
Jun 6, 2001, 11:44:12 AM6/6/01
to

Vincent Maycock wrote:

> Alan wrote in message <3B0D0FB7...@mac.com>...
> >Is there a theory states a photon is a particle when interacting with an
> >electron but a wave when not? If there is not, why?
>
> If we have some way of knowing where the photon is, it will behave like a
> particle. The only way we have of knowing where a photon is, is to have it
> interact with something, though. So in all "interactions," a photon behaves
> like a particle. In between interactions, it behaves like a wave. What's
> really being wave-like is the probability that the photon will interact with
> something at a given point in space.
>
> --
> Vince

Not quite true, since a photon only ever behaves like a "particle" when it is
observed by some measuring apparatus. Otherwise, photons would have to interact
with other photons like "particles" and we'd never see any interference
effects. In fact, the theory that Alan is looking for is Quantum Field Theory
in which all photons in the universe are described as a quantised excitation of
the Electromagnetic field. This provides a unified picture and removes any
considerations of wave-particle duality that occur in non-reletavistic quantum
mechanics.

Matt

Jim Carr

unread,
Jun 8, 2001, 9:23:06 PM6/8/01
to
In article <3b0eb0d1$1...@news2.vip.uk.com>,
c.h.thompson <c.h.th...@newscientist.net> writes
}
} Since it has the characteristics of a wave, why should it not
} disperse like a wave?

In article <Nrrw+lAz...@clef.demon.co.uk>
Charles Francis <cha...@clef.demon.co.uk> writes:
>
>If you do not know where a particle is going, you need a formalism that
>describes the possibility that it goes in different direction.

And, it is worth pointing out, there are soliton waves that do
not disperse just as there are quantum wave packets that do.

>Only the formalism behaves like waves.

And not real ones (in the real/imaginary/complex sense).

>It is always a particle when it is
>confined. Of course it never is perfectly combined, and nor do we know
>precisely the mechanisms in the apparatus used to confine it. So this
>lack of knowledge also requires a wave in the formalism. You continue to
>confuse our lack of complete knowledge with reality. Waves only describe
>our lack of knowledge.

That is, I might add, the Bayesian interpretation of QM that
is based on replacing the assumption that all probability
amplitudes are real with one that allows them to be complex
and associates "collapse" with new information.

--
James Carr <j...@scri.fsu.edu> http://www.scri.fsu.edu/~jac/

spammer fodder: ab...@aol.com ab...@yahoo.com ab...@hotmail.com
ab...@msn.com ab...@earthlink.com

Charles Francis

unread,
Jun 10, 2001, 11:42:41 AM6/10/01
to
In article <9frtpq$q1k$1...@news.fsu.edu>, Jim Carr
<j...@dirac.csit.fsu.edu> writes

>In article <3b0eb0d1$1...@news2.vip.uk.com>,
>c.h.thompson <c.h.th...@newscientist.net> writes
>}
>} Since it has the characteristics of a wave, why should it not
>} disperse like a wave?
>
>In article <Nrrw+lAz...@clef.demon.co.uk>
>Charles Francis <cha...@clef.demon.co.uk> writes:
>>
>>If you do not know where a particle is going, you need a formalism that
>>describes the possibility that it goes in different direction.
>
> And, it is worth pointing out, there are soliton waves that do
> not disperse just as there are quantum wave packets that do.
>
>>Only the formalism behaves like waves.
>
> And not real ones (in the real/imaginary/complex sense).
>
>>It is always a particle when it is
>>confined. Of course it never is perfectly combined, and nor do we know
>>precisely the mechanisms in the apparatus used to confine it. So this
>>lack of knowledge also requires a wave in the formalism. You continue to
>>confuse our lack of complete knowledge with reality. Waves only describe
>>our lack of knowledge.
>
> That is, I might add, the Bayesian interpretation of QM that
> is based on replacing the assumption that all probability
> amplitudes are real with one that allows them to be complex
> and associates "collapse" with new information.
>
Bayesian interpretation is, I think, a modern name for it. There is a
family of interpretations, which I call information theoretic
interpretations, including Von Neumann, Mermin, Youssef, etc which
express this idea in different ways. This goes right back to the
Copenhagen interpretation as described in Physics and Philosophy, where
Heisenberg eschews Bohr's notion of complementarity and says QM
describes what we know of a state, not the state itself.


Regards

--
Charles Francis

Charles Francis

unread,
Jun 12, 2001, 6:08:01 AM6/12/01
to
In article <3B0D0FB7...@mac.com>, Alan <kd7...@mac.com> writes

>Is there a theory states a photon is a particle when interacting with an
>electron but a wave when not? If there is not, why?

This is a fundamental tenet of the Copenhagen interpretation according
to Bohr. But it is more complicated than that. For one thing the
electron itself is behaves like a wave, and the photon has the wave
property of the electron when it interacts with it.

I don't think the wave is real. Photons and electrons are both
particles, and that is how they are always observed. But there are times
when we have very little information about them, and then our
information behaves like a wave.

Regards

--
Charles Francis

Charles Francis

unread,
Jun 12, 2001, 6:03:04 AM6/12/01
to
In article <d5nU6OAxV5I7Ewl$@clef.demon.co.uk>, Charles Francis
<cha...@clef.demon.co.uk> writes

>In article <9frtpq$q1k$1...@news.fsu.edu>, Jim Carr
><j...@dirac.csit.fsu.edu> writes
>>
>> That is, I might add, the Bayesian interpretation of QM that
>> is based on replacing the assumption that all probability
>> amplitudes are real with one that allows them to be complex
>> and associates "collapse" with new information.
>>
>Bayesian interpretation is, I think, a modern name for it. There is a
>family of interpretations, which I call information theoretic
>interpretations, including Von Neumann, Mermin, Youssef, etc which
>express this idea in different ways. This goes right back to the
>Copenhagen interpretation as described in Physics and Philosophy, where
>Heisenberg eschews Bohr's notion of complementarity and says QM
>describes what we know of a state, not the state itself.

I might well add that this interpretation has a number of unique
features, which give it a claim to be the correct interpretation over
and above any other. It is usually criticised because, on its own, it is
not really an interpretation at all. If quantum mechanics only tells us
about information, then it does not tell us what the information is
*about*. But this is for me an enabling argument. On its own if qm is
rules of information then this information could be about almost any
metaphysic. In particular qm does not deny genuinely corpuscular realist
models, since the waves appear in the information, not in the ontology.
Any arguments that the standard model is a theory of matter fields, not
of particles must therefore depend on some far deeper facet of qft, and
have nothing do qm itself.

In practice such arguments depend upon the defineability, or otherwise,
of a qft. But I found no such problem with a discrete model, and the
continuum case is also resolved by Scharf and others. In spite of the
fact that Scharf only considers mathematical structure, not metaphysical
interpretation, if you bring this interpretation in to his mathematics
it all makes sense, and there are no divergences.

Secondly in this interpretation qm is at the very least compatible with
gtr, and perhaps even unified, despite the common mythology to the
contrary. If QM is about information, then it is natural to formulate
Hilbert space in a finite region of space-time in which we are able to
carry out the experiments to obtain the fore and after measurement
results from which we obtain our information. So long as this finite
region is not too far off flat, qm can be formulated in it with curved
metric. The remaining problems with compatibility, such as definition of
the stress-energy operator, are simply a matter of normal ordering in
Scharf's finite model as well as my discrete one.


Regards

--
Charles Francis

Uncle Al

unread,
Jun 12, 2001, 2:21:28 PM6/12/01
to

Read 'em and weep, Sunshine
http://www.quantum.univie.ac.at/research/c60/

Despite Buckminsterfullerene's de Broglie wavelength being 400
times msaller than its molecular radius, and having a mass some
1.35 million times that of an electron, it diffracts through a
grating by the book. Not your boook, of course, a real book.

http://www.mazepath.com/uncleal/sunshine.jpg

--
Uncle Al
http://www.mazepath.com/uncleal/
http://www.ultra.net.au/~wisby/uncleal/
(Toxic URLs! Unsafe for children and most mammals)
"Quis custodiet ipsos custodes?" The Net!

Charles Francis

unread,
Jun 12, 2001, 2:49:12 PM6/12/01
to
In article <3B265DA8...@hate.spam.net>, Uncle Al
<Uncl...@hate.spam.net> writes

>Charles Francis wrote:
>>
>> In article <3B0D0FB7...@mac.com>, Alan <kd7...@mac.com> writes
>> >Is there a theory states a photon is a particle when interacting with an
>> >electron but a wave when not? If there is not, why?
>>
>> This is a fundamental tenet of the Copenhagen interpretation according
>> to Bohr. But it is more complicated than that. For one thing the
>> electron itself is behaves like a wave, and the photon has the wave
>> property of the electron when it interacts with it.
>>
>> I don't think the wave is real. Photons and electrons are both
>> particles, and that is how they are always observed. But there are times
>> when we have very little information about them, and then our
>> information behaves like a wave.
>
>Read 'em and weep, Sunshine
>http://www.quantum.univie.ac.at/research/c60/
>
>Despite Buckminsterfullerene's de Broglie wavelength being 400
>times msaller than its molecular radius, and having a mass some
>1.35 million times that of an electron, it diffracts through a
>grating by the book. Not your boook, of course, a real book.
>
Yes. The same rules apply to information about everything. This merely
lends credence to the notion that the same rules apply and even a
macroscopic particle would diffract if you could do the experiment
correctly.


Regards

--
Charles Francis

Douglas Eagleson

unread,
Jun 12, 2001, 6:50:37 PM6/12/01
to

Uncle Al wrote:

> Read 'em and weep, Sunshine
> http://www.quantum.univie.ac.at/research/c60/
>
> Despite Buckminsterfullerene's de Broglie wavelength being 400
> times msaller than its molecular radius, and having a mass some
> 1.35 million times that of an electron, it diffracts through a
> grating by the book. Not your boook, of course, a real book.
>
> http://www.mazepath.com/uncleal/sunshine.jpg
>
> --
> Uncle Al

Al

The other guy was talking in a general fashion. It's all just a system
logic. Without the grating system included in any formulation.

What is the quantum mechanical interpretation of the grating diffraction
system?
It does not exist because the scattering science is classical charge
conception.
This is just a comment, no reply is necessary.

Douglas Eagleson
ll...@lycos.com


Jim Carr

unread,
Jun 12, 2001, 11:24:02 PM6/12/01
to
"franz heymann" <franz....@care4free.net> wrote
in message news:<3b152d4b$0$15030$cc9e...@news.dial.pipex.com>...
}
} Aleksandr Timofeev <a_n_ti...@my-deja.com> wrote in message
} news:e16a4a22.01053...@posting.google.com...
} >
} [Snip all]
}
} Good luck to you. If you had said what you wanted to say in a
} tenth of the number of words you used, I might have read it.
} Life is too short to cope with that amount of verbiage.

In article <e16a4a22.01053...@posting.google.com>
a_n_ti...@my-deja.com (Aleksandr Timofeev) writes:
>
> Well, we shall continue

I am sure you will, ignoring past corrections as well it seems.

> The photon is mathematical abstraction, which in the implicit
>(latent) form reflects existence of discrete power levels in

>microsystems ...

Who cares, when in the explicit form in which it was first
derived by Einstein (and in which it appears in QED) it is
not a reflection of discrete levels in atoms. That is what
matters if you wish to discuss photons, not some strange
verbiage you have made up.

> ... and as a corollary a capability of exchange by

>electromagnetic energy between systems only by discrete portions.

Exchange of energy. And not a corollary. You appear to be
trying to describe some other theory but manage to not do so.

> You never can provick, that the microsystem swallows or radiates
>electromagnetic energy as particles of photons.

I can't figure this out, but you seem to be trying to dismiss
the challenge of explaining Compton scattering from free particles
(which do not have "discrete power levels") without actually
doing so. A good evasion, but evasion is not allowed.

<... snip long exposition not about physics ...>

> From point of view of the "honour" solution for the n-body problem:

Quite unrelated to the photon and Compton scattering.

<... snip lots more not about this physics problem ...>

>. THE SYMMETRY INSIDE THE SOLAR SYSTEM

Also not about the photon.

<... snip even more not about this physics problem ...>

Still no explanation of Compton scattering without a photon.

nightbat

unread,
Jun 13, 2001, 5:14:15 AM6/13/01
to
nightbat wrote

And David Bohm postulated that electrons were classical
particles and quantum waves with almost an intelligence of implicate
order called quantum potential. After the infamous McCarthy debacle,
Einstein had requested that Bohm be allowed to be his assistant but
Princeton refused to renew his contract and so he never taught in the US
again. A very sad loss for the US, but not for physics.


the nightbat

Douglas Eagleson

unread,
Jun 13, 2001, 2:18:54 PM6/13/01
to

Jim Carr wrote:

> I can't figure this out, but you seem to be trying to dismiss
> the challenge of explaining Compton scattering from free particles
> (which do not have "discrete power levels") without actually
> doing so. A good evasion, but evasion is not allowed.
>

> Quite unrelated to the photon and Compton scattering.

> Also not about the photon.
>


> Still no explanation of Compton scattering without a photon.

Free-Particles, have discrete quantum states in quantum mechanics when
scattering a photon. Quantum Mechanics has a problem with the interaction
probability
of a photon with the Free-Particle. Why, did the photon bother to interact
with the
particle? Most people think it is because of a natural probability,
characteristic of the
particle.

What is the problem here with the Free-Particle? Mathematically it is not
possible
to have a less than one probability in this system. For some reason it is
always One
when you see it occur? Particles that do not interact are assumed to have
no
photon of sufficient characteristic to interact with.

What can you say about this? It is fundamental to understanding the concept

of generalizing the observant variables in theoretical physics.

Timofeev is conceptualizing the basis for the Copenhagen Interpretation.
His point
is revolutionary. Why does Minkowski Space Exist? It is answered by the
posting, explaining INFORMATION reconstruction in an interference system
where the causality ENDED at the SLOTS.

>>>>>>>>>>>>>>>>>>Classified work follows>>>>>>>>>>>>>>>>


Douglas Eagleson
ll...@lycos.com

Douglas Eagleson

unread,
Jun 13, 2001, 11:51:48 PM6/13/01
to

Douglas Eagleson wrote:

A Note to Quantum Mechanicist's:


What follow this set of paragraphs is an example of the interpretation
of Quantum Mechanics by someone willing to consider the photon a
logically abstract object. The system of describing physics is simply
elegent when the Copenhagen Probability of Interaction concept is seen to be
a result of the Principle of Relativity's need for a generalized
observer. Our eyes are on example of such an observer.

Please think it to be the same problem. How do our eyes detect and
transform the quanta to information? They are one in the same
questions. You must answer this to study Quantum Mechanics, otherwise
you lose perspective of real proofs of the below system.

After Treeforv's reposting below there follows a logic to implement the
reconstruction of Information obtained from Minkowski Space.
This space is the only place photon's exist, and is only inferred.
There is a little translation problem in Treeforv's posting that
you are not taking account of. When photons do not exist, he means
photons are inferred in his Interpretation of Quantum Mechanics.
Copenhagen does not even address this simple logical beginning
of an intepretation's complete statement.


************************************************************************
The following was posted by Aleksandr Treeforv
on the sci.physic group

************************************************************************
You can play with a virtual radio interferometer.

If you will manage to explain a principle of operation of this type
of interferometer from the photon point of view, then:

I shall believe in existence of photons, and I shall eat my tie or hat.
---------------------------------------

I think, that your navel will be untied, but you can not explain
----------------------------------------------------------------
a principle of operation of a virtual interferometer from a photon
point of view!


Give an evaluation to the size of a photon if the photon has
a wavelength 3.5 cm. and if the distance between antennas is equal
to diameter of globe. :O) !

Any attempts to explain the principle of operation of the given
type of the radio interferometer from a photon point of view
will suffer a fall.

The radio interferometer with independent writing of signals on
"slots" is a direct proof of a non-existence of photons in a nature.

By my former scientific chief Matveenko Leonid Ivanovich (he works
at the Space Research Institute RAS till now) in 1963 was invented
the absolutely new kind of the radio interferometer.

Main ideas of this type of the radio interferometer were:
1. A simultaneous independent recording of signals on each
separate antenna ("slot") on magnetic tapes;
2. " The interference picture " is received in the computer
as an outcome _ mathematical _ addings of signals recorded on
magnetic tapes;
3. The distance between antennas ("slots ") of a radio
interferometer can exceed diameter of the Earth. (For definit
wave length limiting distances between antennas, at which the
interference picture disappears, is not known until now!)

There are no problems for explanation of a principle of operation
of the radio interferometer with simultaneous independent writing
of signals from a wave point of view.

---------------------
Here for the first time clearly emerges, that for a hypothetical
particle of a photon there is no necessity to pass simultaneously
through both slots (antennas), since the virtual interference
abstractly or mathematically will be realized in the computer at
any convenient time hereafter. !!! It is the experimental fact!!!

How the admirers of a hypothesis of photons now will explain
an interference?
---------------------

But any attempts to explain a principle of operation of the given
type of the radio interferometer from a photon point of view will
suffer a fall for the following reasons:

1. In this case there is no real physical process of an
interference - interference will be realized abstractly
mathematically in the computer;
2. Give an evaluation to the size of a photon if the photon has
a wavelength 3.5 cm. and if the distance between antennas is equal
to diameter of globe. Average on space density of energy impresses,
the delay of time in all processes impress too.


The quantum microsystems can absorb energy only by quantum
portions. This energy is absorbed as electromagnetic waves by
quantum microsystems at random coincidence of orientation of a
spatial dynamic configuration of a quantum microsystem with
orientation of an electromagnetic wave. Analogy between a quantum
microsystem and directional antenna here is conducted in an
obvious kind.

-------------------------------------------------------------------------
From: Aleksandr Timofeev (t...@alpha.dnttm.rssi.ru)
Subject: PHOTONS DO NOT EXIST IN A NATURE
Newsgroups: sci.physics.particle
Date: 1999/01/19

On my sight, for consideration of an offered problem from all points of
view, the most approaching measuring instrument is the microwave interferometr
with superlong basis. I would name this type of an interferometer as an
interferometer with independent registration of signals in shoulders.
Fundamentally any other interferometer by nothing differs from
an interferometer considered below.

Principles of work.

The microwave interferometr with superlong basis consists of two radio
telescopes were on a very large distance from each other. Before experiment
or after him, the nuclear hours are synchronized. Each radio telescope writes
on a videotape a transformed radiation accepted by an antenna. Simultaneously
with a signal, the scores of time received from the standard of frequency,
are written on a videotape.
After ending experiment we have two videotapes with entries of a signal
and scores of time. The "interference picture" is received after data
processing of these videotapes on the computer.

There are two graphic schemes illustrating the description:

The microwave interferometer with superlong basis. Part 1.
Block scheme.

-> radio-telescope 1
->
-> parabolic antenna tape 1 clock 1
-> \
-> \ [ microwave ]
-> \ [ receiver + ] [videotape] [hydrogen ]
-> ) )--->[analog-to-digital]--->[recorder ]<---[frequency]
-> / [ converter ] ^ ^ [standard ]
-> / | |
-> / radio-signals time-marks
-> microwave
-> radiation
-> for synchronization of atomic clocks
-> [transportable caesium]
-> [ frequency standard ]
[snip] ====================================================================
-> radio-telescope 2
->
->
-> parabolic antenna 2 tape 2 clock 2
-> \
-> \ [ microwave ]
-> \ [ receiver + ] [videotape] [hydrogen ]
-> ) )--->[analog-to-digital]--->[recorder ]<---[frequency]
-> / [ converter ] ^ ^ [standard ]
-> / | |
-> / radio-signals time-marks
->
->
->
The microwave interferometer with superlong basis. Part 2.
----------------------------------------------------------

"Interference picture"
^
|
[videotape 1] ------> [ COMPUTER ] <---------- [videotape 2]
^ ^
| |
radio-telescope 1 <- synchronization clocks -> radio-telescope 2
Length of basis
|<----------------------------- {snip} ------------------------------->|
/^\ /^\

^ ^ ^ ^ ^ ^ ^ ^ ^ ^ ^ ^ {snip} ^ ^ ^ ^ ^ ^ ^ ^ ^ ^ ^
| | | | | | | | | | | | ...... | | | | | | | | | | |
Noise microwave radiation


Flexible possibilities of a computer interference of signals.

1. Our interferometer has the right and left shoulders. The distance
between shoulders does not influence sensitivity of an interferometer. The
sensitivity of an interferometer to a signal is determined by the worse
receiver from both radio telescopes.
- The distance between shoulders of an interferometer can be no matter
how large. (This problem is reduced to a problem of transportation of clocks
of a synchronization).
2. The addition of signals is carried out in the computer, that allows
to apply no matter how complicated algorithms of addition of signals.
- In that specific case, we can arbitrary vary delay of signals in each
from a shoulders in any direction.

Conditionality of physical concept " an Interference picture ".

Here we shall be convinced of a celebration of a principle of a causality.
The events happening on slots of an interferometer have primary significance,
all other events happening in an interferometer have the status
secondary.
Let's analyze physical concept addition of signals in an interferometer.
The radiation incident on an input of an interferometer has the following
characteristics:
Wavefront; Frequency band; Spectral fluence of energy;
For each frequency:
Polarization; Amplitude; Phase; Stability.
The interferometer considered by us, is an interferometer with
independent registration of signals in shoulders and the process of addition
of signals is carried out in the computer. The phrase " process of addition
of signals is carried out in the computer " allows clearly to seize essence "
concepts of an interference picture " and source of an origin of this
concept. In the given type of an interferometer there is some arbitrariness
in choice by us of the law of addition of signals from the right and left
shoulders. In our case " the kind of an interference picture " depends on
the concrete law of addition of signals selected by us. In other kinds of
interferometers geometry (physical) construction of an interferometer
determines the law of addition of signals and " a kind of an interference
picture ".
Conclusion 1. For existence of the phenomenon of an interference the
first necessary condition is the registration of a signal for two sites (two
orifices) of a wavefront, which are located on some distance from each other.
Conclusion 2. For existence of the phenomenon of an interference the
second necessary condition is the filtration of frequency band of accepted
signals. The band of a signal should decrease with a diminution of a distance
between two sites (radiation flux) of a wavefront and on the contrary. The
stability of a source of a signal plays the role also.
Conclusion 3. For existence of the phenomenon of an interference the
third necessary condition is the sufficient total energy of an accepted signal
or it is necessary to accumulate a signal.
Conclusion 4. For existence of the phenomenon of an interference the fourth
necessary condition is the existence of the adder of signals from the right
and left shoulders.

Major parameters of an interferometer.

Give is summable stated above. Major parameters of an interferometer are:
Distance between antennas (slots);
Frequency band of registered signals;
Sensitivity of an interferometer.


Sensitivity of an interferometer.

The interferometer represents the macroscopic system.
What determines sensitivity of an interferometer? The effective square of
an antenna determines a stream of energy of a signal, than more square of an
antenna by that the greater stream of energy acts on an input of the
amplifier of a signal. The first cascade of the amplifier of a signal
determines quality of the amplifier. It represents the macroscopic system. If
the macroscopic system is in a thermodynamic equilibrium, it's the state is
characterized in temperature.
This temperature characterizes own noise of the amplifier. Energia of a
signal should exceed energy of own noise of the amplifier, therefore own
noise of the amplifier set threshold energy of a signal, which determines
sensitivity of an interferometer.
The minimum energy stream discovered by an interferometer is close
to 3*10^-21 wt/m^2, that is equivalent to several tens photons on a
square meter per one second (this value requires multiplication on square of
an antenna).

Conclusions.

1. In a considered microwave interferometer, the properties of a rather
sparse stream of absorbed photons are registered. (What is the time of life of
"photon" absorbed by an antenna?). Interferometer has "a very large mass".
2. Varying delay of time of a signal in the certain shoulder of an
interferometer, we define stability of frequency of a source in a direction
of a falling stream. The similar experiments frequently were carried out in
optics.
3. A new information we could receive increasing a distance between
radio telescopes. We could clarify on what distance on a wavefront the
correlation disappears (if disappears). The similar experience are not known
to me. For basis equal to the Earth's diameter, the correlation reliably
exists. The resolving power of the radio interferometer approximately in
100 - 1000 times is exceeded with a resolving power of an optical telescope.
4. We shall name photons, in a stream incident on an antenna, " as free
photons ". Let's name photons absorbed by an antenna, " as bound photons ".
Whether we know a structure " of bound photons "? To me it is not clear. I
definitely know, that the structure of an electromagnetic field is determined
by boundary conditions. For this reason the structure " of free photons
" should differ from a structure " of bound photons ". The detailses of


mechanisms of a radiation and absorption of an electromagnetic field are not
known to us. On my sight the photon is simply other title for mechanisms of
a radiation and absorption of an electromagnetic field. For this reason the

PHOTONS DO NOT EXIST IN A NATURE or, if it is pleasant more to you, in


a medium of an electromagnetic field the photons will be generated as virtual
particles.

We shall come to a conclusion:
The photons are particles - ghosts, the photons are mathematical
abstraction, which allow us to calculate probability of interaction of an
electromagnetic field and substance.

***********************************************************************
End of Treeforv posting
***********************************************************************


What follows is a Java program that implements the logic explained in the
beginning paragraph. Do not look at it as an algorithm, the output logic must
be related to the Functional State Logic supplied by the user. In order to use
it properly the eigenfunctions of the Minkowski Space must be assumed to be
UNKNOWNS, to be discovered by trial and error. The test for a valid solution
is then performed by a Image Recognition algorithm. A separate algorithm MUST
be used because the INTEGER FUNCTION Logic is a Mathematical Operation
of Fundamental type. It is very hard to elucidate, but is a real logic
of nature like Division. It is likely to be the same logic used by
the human optic nerve for optical transformation. You might want to
consider constructing a nice fast analog computer to implement the
INTEGER FUNCTION.

The example below is one dimensional, but is easily modified to
a four-dimensional Hamiltonian solution FUNCTION.


#######Warning the below program looks deceptively simple, but the outcome
logic is unique and not definable using number theory. You must
experiment with it to see it's elegance.##################

***********************************************************************

/*
The mathematical logic implemented by this class, comprises the concept of generalized
matrix mathematics. The term shuffling function is used to refer to this basic logical
operation
of integer matrix mathematics. It is a logic very ammenable to the creation of linguistic

definitions and quantum mechanical definitions.

Please note that generalized mathematical matrix operations are inclusive of the standard
arithmatic operations(+,-,/,*). Shuffling is a fundamental logic of symbolic manipulation

and can implement the standard operations-with significant programming effort. This is
a CLASS of mathematical operation NOT SIMPLE LIKE ADDITION.

The integer function in this example was intended to be a constant = 1, except an
encryption
key was included for another reason, to make three unique shuffles for the GPS
application,
and creates an unelucidatable, but wholly definable function. To make the function, i.e.
the logic
applied to the shuffle, be a unit constant, "the key" must be set to 1. And several
nested loops
must be removed, also, leaving a single loop. Be warned- This program is not for
encryption
use, it is a very simple function.

***The key to thinking in such a general symbolic way is to realise that the range of a
continous
integer function must always be explicitly included in the matrice's definition.*** The
logic
of the function is then reducable to a set of matrix shuffling operation boundaries.

Given this concept, this mathematics may implement the Calculus.

Note: This is a non-standard mathematics. P.S. The Integral must not use a graphical
interpretation and functional continuity must be defined without a spatial continuity
concept.
Philosophically, mathematics are purely descriptions of our error-prone observations.
An integer is defined to include observation results in decimal point format. Significant

digits and unit analysis are real controling factors in any system.
*/

// THIS java class IS INCLUDED AS INFORMATION ONLY WITH THE moddes ENCRYPTION PROGRAM.

public class Logic {

int h;
int i;
int j;
int k;
int l;
int m;
int n;
int o;
int p;
int q;
int r;
int Z[] = new int[9];
int arr[] = new int[59];
int A[] = new int[59];
int Key[] = new int[9];
int iterend = 9; // this defines functional logic also. This is non-standard symbolic
logic.
int fix; //selects one of the key values

public Logic () {}

public int[] shuffle ( int A[], int key[][], int fix ) {

for ( i=0; i<9; i++){
Key[i]= key[fix][i];
} //load key locally

for ( i=0; i<59; i++){
A[i]=A[i];
}


for ( h=0; h<1; h++){ //this controls the degree of shuffling, a good shuffle has h=1000

//start of shuffle logic, REMEMBER, IT IS THE LOGIC OF THIS CLASSES OUTCOME THAT IS
UNIQUE.

for ( i=0; i<59; i++){

for( j = Key[0]; j<iterend; j++ ) {
if( (i-j*9) >= 0 ){
Z[0] = A[i-j*9];
A[i-j*9]= A[i];
A[i] = Z[0];
}

for( k = Key[1]; k<iterend; k++ ) {
if( (i-k*9) >= 0 ){
Z[1]= A[i-k*9];
A[i-k*9]= A[i];
A[i] = Z[1];
}

for( l = Key[2]; l<iterend; l++ ) {
if( (i-l*9) >= 0 ){
Z[2]= A[i-l*9];
A[i-l*9]= A[i];
A[i] = Z[2];
}

for( m = Key[3]; m<iterend; m++ ) {
if( (i-m*9) >= 0 ){
Z[3]= A[i-m*9];
A[i-m*9]= A[i];
A[i] = Z[3];
}

for( n = Key[4]; n<iterend; n++ ) {
if( (i-n*9) >= 0 ){
Z[4]= A[i-n*9];
A[i-n*9]= A[i];
A[i] = Z[4];
}

for( o = Key[5]; o<iterend; o++ ) {
if( (i-o*9) >= 0 ){
Z[5]= A[i-o*9];
A[i-o*9]= A[i];
A[i] = Z[5];
}

for( p = Key[6]; p<iterend; p++ ) {
if( (i-p*9) >= 0 ){
Z[6]= A[i-p*9];
A[i-p*9]= A[i];
A[i] = Z[6];
}

for( q = Key[7]; q<iterend; q++ ) {
if( (i-q*9) >= 0 ){
Z[7]= A[i-q*9];
A[i-q*9]= A[i];
A[i] = Z[7];
}

for( r = Key[8]; r<iterend; r++ ) {
if( (i-r*9) >= 0 ){
Z[8]= A[i-r*9];
A[i-r*9]= A[i];
A[i] = Z[8];
}

}}}}}}}}}; //nested loops

} //end of inner step loop , end of shuffle definition

} // end big loop, this partially defines the particular integer function

for(j=0;j<59;j++) {
arr[j]=A[j];
}
return arr;
} //end method

} // end of class

*************************************************************************

Douglas Eagleson
ll...@lycos.com


Jim Carr

unread,
Jun 24, 2001, 1:23:29 AM6/24/01
to
In article <LOMhHMCY...@clef.demon.co.uk>
Charles Francis <cha...@clef.demon.co.uk> writes:
>
<... snip lots of stuff I generally agree with ...>

>
>Secondly in this interpretation qm is at the very least compatible with
>gtr, and perhaps even unified, despite the common mythology to the
>contrary.

Exactly my thoughts on the matter (and a point Saul hinted at
in his first paper), but I have not tried to solve what I consider
to be a relevant problem within that scheme. However, I think
it can be done.

e-mail info: new...@fbi.gov pyr...@ftc.gov enfor...@sec.gov
ab...@aol.com ab...@yahoo.com ab...@hotmail.com

Jim Carr

unread,
Jun 25, 2001, 10:23:17 PM6/25/01
to

Jim Carr wrote in <9g6mci$cfp$1...@news.fsu.edu>:
|
<... much snipped by Douglas ...>

|
| In article <e16a4a22.01053...@posting.google.com>
| a_n_ti...@my-deja.com (Aleksandr Timofeev) writes:
| > ... and as a corollary a capability of exchange by
| >electromagnetic energy between systems only by discrete portions.
|
| Exchange of energy. And not a corollary. You appear to be
| trying to describe some other theory but manage to not do so.
|
| > You never can provick, that the microsystem swallows or radiates
| >electromagnetic energy as particles of photons.
|
| I can't figure this out, but you seem to be trying to dismiss
| the challenge of explaining Compton scattering from free particles
| (which do not have "discrete power levels") without actually
| doing so. A good evasion, but evasion is not allowed.
|
| <... snip long exposition not about physics ...>
|
| > From point of view of the "honour" solution for the n-body problem:
|
| Quite unrelated to the photon and Compton scattering.
|
| <... snip lots more not about this physics problem ...>
|
| >. THE SYMMETRY INSIDE THE SOLAR SYSTEM
|
| Also not about the photon. ...

In article <3B27AE8D...@lycos.com>

Douglas Eagleson <ll...@lycos.com> writes:
>
>Free-Particles, have discrete quantum states in quantum mechanics when
>scattering a photon.

Free particles either have discrete quantum states or they do not.
The electron does not have any excited states, so as a free particle
it only has continuum states in quantum mechanics.

I have not idea where you got that idea about a free particle
having discrete states.

>Quantum Mechanics has a problem with the interaction probability
>of a photon with the Free-Particle.

No it does not.

>Why, did the photon bother to interact with the particle?

Because it had charge.

>Most people think it is because of a natural probability,
>characteristic of the particle.

I have no idea where you get that idea. The coupling is to charge.

>What is the problem here with the Free-Particle?

None, unless you try to explain Compton scattering without
using the photon.

>Mathematically it is not possible
>to have a less than one probability in this system.

Look at the angular distribution. It is not equal to unity at
all angles.

>What can you say about this?

At some point I had to simply snip the rest because it makes even
less sense than the first part.

e-mail info: new...@fbi.gov pyr...@ftc.gov enfor...@sec.gov
ab...@aol.com ab...@yahoo.com ab...@hotmail.com

Aleksandr Timofeev

unread,
Jul 19, 2001, 7:05:23 AM7/19/01
to
j...@dirac.csit.fsu.edu (Jim Carr) wrote in message news:<9h8rml$itr$1...@news.fsu.edu>...

> Jim Carr wrote in <9g6mci$cfp$1...@news.fsu.edu>:
> |
> <... much snipped by Douglas ...>
> |
> | In article <e16a4a22.01053...@posting.google.com>
> | a_n_ti...@my-deja.com (Aleksandr Timofeev) writes:
> | > ... and as a corollary a capability of exchange by
> | >electromagnetic energy between systems only by discrete portions.
> |
> | Exchange of energy. And not a corollary. You appear to be
> | trying to describe some other theory but manage to not do so.
> |
> | > You never can provick, that the microsystem swallows or radiates
> | >electromagnetic energy as particles of photons.
> |
> | I can't figure this out, but you seem to be trying to dismiss
> | the challenge of explaining Compton scattering from free particles
> | (which do not have "discrete power levels") without actually
> | doing so. A good evasion, but evasion is not allowed.
> |

Dear Jim Carr, be kind, give the references to Internet sites
circumscribing Compton scattering from free electrons. ;o)

<... snip long exposition not about Compton scattering
from free electrons ...>

Aleksandr Timofeev

Aleksandr Timofeev

unread,
Jul 19, 2001, 12:22:49 PM7/19/01
to

Aleksandr Timofeev

------------------------------------------------------------
Get your FREE web-based e-mail and newsgroup access at:
http://MailAndNews.com
------------------------------------------------------------

Eric Prebys

unread,
Jul 19, 2001, 3:00:19 PM7/19/01
to

Aleksandr Timofeev wrote:
>
> j...@dirac.csit.fsu.edu (Jim Carr) wrote in message
> news:<9h8rml$itr$1...@news.fsu.edu>...
> > Jim Carr wrote in <9g6mci$cfp$1...@news.fsu.edu>:
> > |
> > <... much snipped by Douglas ...>
> > |
> > | In article <e16a4a22.01053...@posting.google.com>
> > | a_n_ti...@my-deja.com (Aleksandr Timofeev) writes:
> > | > ... and as a corollary a capability of exchange by
> > | >electromagnetic energy between systems only by discrete portions.
> > |
> > | Exchange of energy. And not a corollary. You appear to be
> > | trying to describe some other theory but manage to not do so.
> > |
> > | > You never can provick, that the microsystem swallows or radiates
> > | >electromagnetic energy as particles of photons.
> > |
> > | I can't figure this out, but you seem to be trying to dismiss
> > | the challenge of explaining Compton scattering from free particles
> > | (which do not have "discrete power levels") without actually
> > | doing so. A good evasion, but evasion is not allowed.
> > |
>
> Dear Jim Carr, be kind, give the references to Internet sites
> circumscribing Compton scattering from free electrons. ;o)
>

I'm not Jim Carr, but Compton scattering off electron beams
has been done for some time. Among other things, it's a standard
technique for measuring beam polarization. As a start, you can
check out the polarimeter for the SLD experiment at the
Stanford Linear Accelerator Center (SLAC).

http://fnlib2.fnal.gov/MARION/AAE-8353

I'm sure there are plenty of supporting references.

I worked on an experiment that observed ordinary and
nonlinear Compton scattering from free electrons, you
can find our publications at our web site

http://www.slac.stanford.edu/exp/e144/e144.html

Since I didn't see your original posting, I don't
know what the point was, but certainly Compton scattering
off of free electrons is a well-established fact.

-Eric


> <... snip long exposition not about Compton scattering
> from free electrons ...>
>
> Aleksandr Timofeev
>
> ------------------------------------------------------------
> Get your FREE web-based e-mail and newsgroup access at:
> http://MailAndNews.com
> ------------------------------------------------------------

--
---------------------------------------------------------
* Eric Prebys, Physics Department, Princeton University *
* 609-258-4910, FAX: -6360, Email: pre...@princeton.edu *
* WWW: http://www.princeton.edu/~prebys/ *
---------------------------------------------------------

Aleksandr Timofeev

unread,
Jul 24, 2001, 10:57:35 AM7/24/01
to
j...@dirac.csit.fsu.edu (Jim Carr) wrote in message news:<9h8rml$itr$1...@news.fsu.edu>...
> Jim Carr wrote in <9g6mci$cfp$1...@news.fsu.edu>:
> |
> <... much snipped by Douglas ...>
> |
> | In article <e16a4a22.01053...@posting.google.com>
> | a_n_ti...@my-deja.com (Aleksandr Timofeev) writes:
> | > ... and as a corollary a capability of exchange by
> | >electromagnetic energy between systems only by discrete portions.
> |
> | Exchange of energy. And not a corollary. You appear to be
> | trying to describe some other theory but manage to not do so.
> |
> | > You never can provick, that the microsystem swallows or radiates
> | >electromagnetic energy as particles of photons.
> |
> | I can't figure this out, but you seem to be trying to dismiss
> | the challenge of explaining Compton scattering from free particles
> | (which do not have "discrete power levels") without actually
> | doing so. A good evasion, but evasion is not allowed.
> |
> | <... snip long exposition not about physics ...>
> |

Dear Jim Carr, be kind, give the references to Internet sites
circumscribing Compton scattering from free electrons for values
of energies characteristic of atomic (not nuclear) physics...


<... snip long exposition not about Compton scattering

from free electrons for usual values of energies characteristic
of atomic (not nuclear) physics... ...>

In the given moment I offer to exclude from consideration over
strong fields and values of speeds of electrons close to speed
of light.

Aleksandr Timofeev

franz heymann

unread,
Jul 24, 2001, 4:03:53 PM7/24/01
to

Aleksandr Timofeev <a_n_ti...@my-deja.com> wrote in message
news:e16a4a22.01072...@posting.google.com...

It sounds as if you should do a Google search on "Thomson
scattering". In the low energy limit, Compton scattering
asymptotes to Thomson scattering

Franz Heymann


Aleksandr Timofeev

unread,
Jul 26, 2001, 11:05:17 AM7/26/01
to
"franz heymann" <franz....@care4free.net> wrote in message news:<3b5de1a7$0$3759$cc9e...@news.dial.pipex.com>...

> Aleksandr Timofeev <a_n_ti...@my-deja.com> wrote in message
> news:e16a4a22.01072...@posting.google.com...
> > j...@dirac.csit.fsu.edu (Jim Carr) wrote in message
> news:<9h8rml$itr$1...@news.fsu.edu>...
> > > Jim Carr wrote in <9g6mci$cfp$1...@news.fsu.edu>:
> > > |
> > > | <... much snipped by Douglas ...>
> > > |
> > > | In article
> > > | <e16a4a22.01053...@posting.google.com>
> > > | a_n_ti...@my-deja.com (Aleksandr Timofeev) writes:
> > > | > ... and as a corollary a capability of exchange by
> > > | >electromagnetic energy between systems only by discrete
> > > | portions.
> > > |
> > > | Exchange of energy. And not a corollary. You appear to be
> > > | trying to describe some other theory but manage to not do
> > > | so.
> > > |
> > > | > You never can provick, that the microsystem swallows
> > > | > or radiates electromagnetic energy as particles - photons.

> > > |
> > > | I can't figure this out, but you seem to be trying to dismiss
> > > | the challenge of explaining Compton scattering from free particles
> > > | (which do not have "discrete power levels") without actually
> > > | doing so. A good evasion, but evasion is not allowed.
> > > |
> > > | <... snip long exposition not about physics ...>
> > > |
> >
> > Dear Jim Carr, be kind, give the references to Internet sites
> > circumscribing Compton scattering from free electrons for values
> > of energies characteristic of atomic (not nuclear) physics...
> >
> >
> > <... snip long exposition not about Compton scattering
> > from free electrons for usual values of energies characteristic
> > of atomic (not nuclear) physics... ...>
> >
> > In the given moment I offer to exclude from consideration over
> > strong fields and values of speeds of electrons close to speed
> > of light.
>
> It sounds as if you should do a Google search on "Thomson
> scattering". In the low energy limit, Compton scattering
> asymptotes to Thomson scattering

I consider, that Compton has accepted error model for scattering
of radiation on "free" electrons. Actually "free" electrons,
investigated by him, belonged to quantum systems. For this reason
the actual process of scattering consists of three phases:

1 phase - the quantum system absorbs a quantum portion of
radiation;
2 phases - quantum system are in a metastable state;
3 phases - the quantum system or radiates an absorbed quantum
portion of radiation or partially breaking up loses (shoot) an electron
and radiating electromagnetic radiation have a smaller wavelength.

Both conceptual approaches have in an eventual result the identical
mathematical formulation, but the concept of "photon" is excessive
for the interpretation of scattering of radiation offered by me.

Aleksandr Timofeev

unread,
Aug 9, 2001, 10:58:18 AM8/9/01
to
Eric Prebys <pre...@princeton.edu> wrote in message news:<3B572E43...@princeton.edu>...

As it will be said in message below:

"the Compton scattering is treated in an entirely classical
way, without using energy and momentum conservation, but just standard
classical em + relativistic *kinematics*, by the picture of a
circularly polarized em wave impinging upon a charged particle.
The calculation is based on deriving a steady-state solution for the
down-stream motion of the particle which is superimposed to the
constant rotation at the frequency of the passing wave."

Comments?

-----------------------------------------------------------------------
(beginning of original message)

From: Fabio Di Teodoro (fdit...@brynmawr.edu)
Subject: Re: Existence of photons
Newsgroups: sci.physics
Date: 1999/03/08

>
> On 8 Mar 1999, Jim Carr wrote:
>
> > In article <7brmkl$fcb$8...@thorn.cc.usm.edu>
> > lrm...@orca.st.usm.edu (Larry Mead) writes:
> > >
> > >The need to quantize electrodynamics was argued thoroughly by Jaynes,
> > >Lamb and lots of quantum optics folks in the 70's. It turns out, there
> > >is one and only one effect which requires this: spontaneous emission.
> >
> > Do you have a reference to an article that treats Compton
> > classically and points out the error in Compton's calculation
> > of the angular distribution?

i guess i now know what you meant, for the first.
You should take a look at

"Atoms and light" by John N. Dodd (Plenum Press, New York, 1991).

In Chapter 6, the Compton scattering is treated in an entirely classical
way, without using energy and momentum conservation, but just standard
classical em + relativistic *kinematics*, by the picture of a
circularly polarized em wave impinging upon a charged particle.
The calculation is based on deriving a steady-state solution for the
down-stream motion of the particle which is superimposed to the
constant rotation at the frequency of the passing wave.
I haven't read the analysis in detail, but my first impression is that
it is quite clever.

It is, especially in light of the comments at page 55, apparent that the
standard Compton effect, i.e. the one the Compton explained using the
notion of photon, does not actually *need* this notion.
So, according to the author, the standard (spin-free) Compton effect
cannot be invoked to argue the existence of photons.
I am sorry if some of you do not have access to this book, unfortunately
the author does not provide other references in which the calculation has
been presented.

That was very educational for me. I confess i did not know about this.

In Ch.12, p.144 the author also presents a critical treatment of the
photoelectric effect. Again, he does not only address the issue of
the "photonless" interpretation of the experiment, but also adds that
some subtle question which, at first sight, *could* lead to a photon
interpretation, actually does not.

I finally suggest the readig of Chapter 14, which contains quite of a
compelling criticism on the way this subject is _taught_.

Now i agree with Mati Meron on the fact that many other experiments that
support the photon picture and the QED exist, of course.
I never thought this was in question.
However, i must note that, at least from what i understood from this
thread, the main concern made by Martin Green was an "educational" one
and, from this point of view, i have frankly to agree with him.

Indeed, i can see in the process of *learning* about this subject, the
student risks to be exposed to some evident misconception.
Later on, she/he will (hopefully) learn more on the subject and gain a
deeper insight. But this does not justify the initial misconception.

The notion of photon is introduced quite early in the intro physics
courses. It may be true that some more accurate book does avoid to
invoke the mere, plain photoelectric effect as the evidence of the
photon's existence (although, honestly, i do know several intro-physics
books which carry this misconception).
However, at this point, i have no problem to recognize that this is the
first time i have read about a classical derivation of the Compton
effect. Honestly, i think that *really* this is not included in any
intro-level book, the ones student read while they are learning physics.
I think they all present, in a more or less open way, the standard Compton
effect as one compelling "demonstration" of the existence of photon and
not as an experiment that "supports" the photon's picture.
Please provide references, in the case my claim happens to be wrong.

Maybe the details of this argument may be of interest only to people who
study history of physics, considering that the photon picture has proven
to be successful in so many applications.
Needless to say, the very Compton effect has aspects that are not
correctly addressed by the semiclassical picture (e.g. effect of the
particle's spin) and lead, for example, to the QED Klein-Nishina
differential cross section as i said elsewhere (i hope at least this does
not have a classical derivation... :-) )

However, why should a student be told, even if it's just the beginning of
her/his studies in physics, that "the photon's existence has been
proved by the mere Compton effect" when it's not the case, in that the
Compton lends itself both to a classical derivation and "corpuscolar"
derivation ?

Is it true that the Compton effect is understood by so many students as a
"test" for the existence of photons ?

Thank you all for your contributions.

best regards

end of original message
-----------------------------------------------------------------------

franz heymann

unread,
Aug 9, 2001, 3:23:24 PM8/9/01
to

Aleksandr Timofeev <a_n_ti...@my-deja.com> wrote in message
news:e16a4a22.01080...@posting.google.com...

Does the calculation to which you refer include a calculation of the
change in the frequency of the em wave which occurs when it interacte
with this free electron?

Franz Heymann

[Snip]

Maurice Barnhill

unread,
Aug 13, 2001, 5:35:58 PM8/13/01
to

I am scheduled to teach senior-level quantum mechanics this fall, so I
thought I had better look at John Dodd's book and amke sure that I
didn't say something false sometime during the fall. I have now read
Chapter 6 in detail, everything short of actually verifying the
algebra. I'm afraid that I disagree with the interpretation above of
the results of the chapter. At first glance the chapter reads that way,
but on close inspection I reach exactly the opposite conclusion. In
fairness to Dodd, I don't think he intended to have this chapter read as
an argument that the Compton Effect does not need Quantum Mechanics. In
the preface he writes "Then, in a pivotal chapter, the interaction with
a free charge is described (the Compton effect); it is shown that, in
order to give agreement with observation, certain quantum rules must be
introduced."

So, what is done in the chapter on the Compton Effect. Dodd undertakes
a classical calculation of the motion of an electron in a
circularly-polarized electromagnetic wave. He seeks a steady-state
solution with the electron moving in a helical fashion, with the
velocity along the axis of the helix being time independent. Such a
solution exists, but no condition is obtained on the velocity. He then
calculates (classically) the radiation given off by the electron due to
its acceleration in the circular part of the helical motion. He now
calculates, classically, the momentum, energy, and angular momentum that
has been transferred to the emitted (scattered) wave. He then uses the
*empirical* Compton-scattering formula to determine the velocity of the
electron along the axis of the helix, getting a value that can be
expressed in terms of Planck's constant. The empirical relation is
where the quantization is introduced.

To back up my interpretation of what he has done, I quote from p. 54:
"The Compton effect has been analyzed by this rather unusual route for a
number of reasons. First of all it establishes the same rule for energy
change in interaction as did Planck, and therefore provides an
alternative entry to quantum physics...." In other words, the observed
Compton effect could have been used to show that quantum effects must
exist.

Now if I had lived at that time in this alternate world and had been
smart enough, I would have looked on this derivation as crying out for a
mechanistic interpretation, explaining why the Compton effect had the
form that forced this one value for the velocity of the electron. And
if I had been really smart, I might have thought of the possibility that
light was carried by little corpuscles and called them photons.



| That was very educational for me. I confess i did not know about this.
|

| ...


|
| Now i agree with Mati Meron on the fact that many other experiments
that
| support the photon picture and the QED exist, of course.
| I never thought this was in question.
| However, i must note that, at least from what i understood from this
| thread, the main concern made by Martin Green was an "educational" one
| and, from this point of view, i have frankly to agree with him.
|

--
Maurice Barnhill, m...@udel.edu
http://www.physics.udel.edu/~barnhill/
Physics Dept., University of Delaware, Newark, DE 19716

Martin Green

unread,
Aug 13, 2001, 8:05:33 PM8/13/01
to

Aleksander Timofeyev wrote:

> | You should take a look at
> |
> | "Atoms and light" by John N. Dodd (Plenum Press, New York, 1991).
> |
> | In Chapter 6, the Compton scattering is treated in an entirely
> classical
> |

way......................(snip)............................................s
o,


> | according to the author, the standard (spin-free) Compton effect
> | cannot be invoked to argue the existence of photons.

Maurice Barnhill replied:

> I am scheduled to teach senior-level quantum mechanics this fall, so I
> thought I had better look at John Dodd's book and amke sure that I
> didn't say something false sometime during the fall. I have now read
> Chapter 6 in detail, everything short of actually verifying the
> algebra. I'm afraid that I disagree with the interpretation above of
> the results of the

chapter.....................(snip).........................
> ...............in other words, the observed


> Compton effect could have been used to show that quantum effects must
> exist.

There is a difference between arguing for the existence of "quantum
effects",
and arguing for the "existence of photons". The real question is whether
the Compton effect can be explained without the existence of photons.
The problem with Compton's argument is that he treats the electron
as a tiny charged sphere, failing to take into account "quantum effects".
Even Dodd's argument, as described in Barnhill's post, does not attempt
to analyze the interaction of a classical e-m wave with a quantum,
Schroedinger-type electron.

It is very hard to give a convincing argument for the existence of photons.

Martin Green
http://www.onforeignsoil.com
Teach yourself Yiddish while you read
the exciting autobiography of Falk Zolf.


Joe Rongen

unread,
Aug 13, 2001, 11:59:27 PM8/13/01
to
"Martin Green" <btes...@home.com> wrote in message
news:hXZd7.7069$Tn1.2...@news1.rdc1.mb.home.com...

Not according to this inexpensive Dover (1994) book:

"X-Ray Diffraction in Crystals, Imperfect Crystals, and Amorphous Bodies."
by, A Guinier. ISBN 0-486-68011-8 Quoted from page 13:

"It is known from wave mechanics that the point electron
of classical theory must be replaced by a smooth distribution
of electric charge. Using the charge of the electron as a unit,
the density p(x) of the cloud at the extremity of the vector x
near the nucleus is p(x) = |phi(x)|^2, where phi(x) is the wave
function satisfying the Schroedinger equation. As in classical
theory, the volume dv carrying a charge pdv scatters a wave whose
amplitude is that scattered by a single electron, multiplied by pdv.
The elementary waves which are scattered by the electronic cloud
interfere, since they are coherent and the total effect can be calculated
by using interference formulas."

He than goes on to show/explain those formulas and much more.....

----------------
Regards, Joe
j...@alpha.to


Martin Green

unread,
Aug 14, 2001, 12:56:10 AM8/14/01
to
Martin Green wrote:

> > The problem with Compton's argument is that he treats the electron
> > as a tiny charged sphere, failing to take into account "quantum
effects".

Joe Rongen replied:

> Not according to this inexpensive Dover (1994) book:
>
> "X-Ray Diffraction in Crystals, Imperfect Crystals, and Amorphous Bodies."
> by, A Guinier. ISBN 0-486-68011-8 Quoted from page 13:
>
> "It is known from wave mechanics that the point electron
> of classical theory must be replaced by a smooth distribution
> of electric charge. Using the charge of the electron as a unit,
> the density p(x) of the cloud at the extremity of the vector x
> near the nucleus is p(x) = |phi(x)|^2, where phi(x) is the wave

> function satisfying the Schroedinger equation.....

We were talking about (at least, I was) Compton's 1919 paper
in which he supposedly shows that the scattering of photons by
electrons is not consistent with classical mechanics. The argument
you quote here refers to the Schroedinger equation, which did
not exist until 1926.

franz heymann

unread,
Aug 14, 2001, 6:50:38 AM8/14/01
to

Martin Green <btes...@home.com> wrote in message
news:hXZd7.7069$Tn1.2...@news1.rdc1.mb.home.com...
>

Not the Schrodinger type. You mean the Dirac type. Schrodinger's
equation is not relativistically invariant. Dirac's equation is. The
Compton effect is properly calculated within this background in, for
instance, Heitler's "Relativistic Quantum Mechanics".

>
> It is very hard to give a convincing argument for the existence of
photons.
>

Franz Heymann


Martin Green

unread,
Aug 14, 2001, 10:56:03 AM8/14/01
to
Martin Green wrote:

> > Even Dodd's argument, as described in Barnhill's post, does not
> attempt
> > to analyze the interaction of a classical e-m wave with a quantum,
> > Schroedinger-type electron.

Franz Heyman replied:

> Not the Schrodinger type. You mean the Dirac type. Schrodinger's
> equation is not relativistically invariant. Dirac's equation is. The
> Compton effect is properly calculated within this background in, for
> instance, Heitler's "Relativistic Quantum Mechanics".

I think you are agreeing with me, in any case. My point was
that the flaw with Compton's argument is that he does
not EVEN take into account the properties of
the Schroedinger electron, let alone the Dirac electron.
His electron is basically a tiny, charged metal sphere.

franz heymann

unread,
Aug 14, 2001, 4:58:47 PM8/14/01
to

Martin Green <btes...@home.com> wrote in message
news:7_ae7.7507$Tn1.2...@news1.rdc1.mb.home.com...

> Martin Green wrote:
>
> > > Even Dodd's argument, as described in Barnhill's post, does not
> > attempt
> > > to analyze the interaction of a classical e-m wave with a
quantum,
> > > Schroedinger-type electron.
>
> Franz Heyman replied:
>
> > Not the Schrodinger type. You mean the Dirac type. Schrodinger's
> > equation is not relativistically invariant. Dirac's equation is.
The
> > Compton effect is properly calculated within this background in,
for
> > instance, Heitler's "Relativistic Quantum Mechanics".
>
> I think you are agreeing with me, in any case. My point was
> that the flaw with Compton's argument is that he does
> not EVEN take into account the properties of
> the Schroedinger electron, let alone the Dirac electron.
> His electron is basically a tiny, charged metal sphere.
>
No quibble

Franz Heymann


Aleksandr Timofeev

unread,
Aug 15, 2001, 7:43:25 AM8/15/01
to
"franz heymann" <franz....@care4free.net> wrote in message news:<3b799787$0$3755$cc9e...@news.dial.pipex.com>...

"The ideas are a daughters of the past and are mothers of the future
and always are bondwomans of time!"
Le Bon. "Psychology of races and peoples", 1895, Paris :

"the Schrodinger type"
"the Dirac type"
"Schrodinger's equation"
"Relativistic Quantum Mechanics"

You should know the proverb of physicists:
" With this "plasma" we shall reach a marasmus ".

I shall paraphrase this proverb as follows: " With the
theoretical description of interaction of these "electrons" and
these "electromagnetic waves" we shall reach a marasmus "
It is the constituent of a problem about plasma.

How your beloved "Relativistic Quantum Mechanics" decides the given
problem? - she is powerless.

=============================================
Let's pass to a fruitful part of the dialogue.

As a whole problem is reduced up to areas (boundaries)
of applicability(feasibility) of physical ideas.

Let us assume that you have an analytical method of obtaining
of the Plank constant on the basis of classical or pceudo-classical
physical representations... (interaction between EM and e ...)

"No quibble"

franz heymann

unread,
Aug 15, 2001, 11:17:21 AM8/15/01
to

Aleksandr Timofeev <a_n_ti...@my-deja.com> wrote in message
news:e16a4a22.01081...@posting.google.com...

Let us rather not assume this, since there is no basis whatsoever for
such an assumption.
>
> "No quibble"

The relevance of this remark escapes me

Franz Heymann


Aleksandr Timofeev

unread,
Aug 16, 2001, 5:40:11 AM8/16/01
to
"franz heymann" <franz....@care4free.net> wrote in message news:<3b738890$0$3762$cc9e...@news.dial.pipex.com>...
I am afraid that we interprete concept " of a free electron "
by completely different ways.
Please put particular examples of physical realizations, in which
you see existence " of free electrons ".

Whether you can give definition of concept " a free electron "?
(i.e. give the description of physical properties " of free electrons ")


> Franz Heymann
>
> [Snip]

Aleksandr Timofeev

unread,
Aug 16, 2001, 5:59:25 AM8/16/01
to
Eric Prebys <pre...@princeton.edu> wrote in message news:<3B572E43...@princeton.edu>...

I am afraid that we interprete concept " of a free electron "

by completely different ways.
Please put particular examples of physical realizations, in which
you see existence " of free electrons ".

Whether you can give definition of concept " a free electron "?
(i.e. give the description of physical properties " of free electrons ")


---
Aleksandr Timofeev

franz heymann

unread,
Aug 16, 2001, 11:10:41 AM8/16/01
to

Aleksandr Timofeev <a_n_ti...@my-deja.com> wrote in message
news:e16a4a22.0108...@posting.google.com...
>
[Snip]

> I am afraid that we interprete concept " of a free electron "
> by completely different ways.

Not to mince words, in that case you have got it wrong and I have lost
interest.


Franz Heymann


Aleksandr Timofeev

unread,
Aug 16, 2001, 11:43:01 AM8/16/01
to
In article <3b7be3bb$0$238$cc9e...@news.dial.pipex.com>, franz heymann says...

You can not find the right answer to a so simple problem: What is " a free
electron "?


Bryan Reed

unread,
Aug 16, 2001, 1:17:58 PM8/16/01
to
In article <9SRe7.3450$2u.3...@www.newsranger.com>,


I forget how many crackpot points it is for confusing boredom with
inabiliity to answer . . .

A free electron is one that can be taken to be moving in empty space, with
(to zeroth order) no externally-applied electric or magnetic fields. So
when you're talking about the interaction of a free electron with an
electromagnetic wave, you're saying the electron is interacting with this
wave and nothing else.

Have fun,

Bryan

Aleksandr Timofeev

unread,
Aug 23, 2001, 8:18:33 AM8/23/01
to
bwr...@u.washington.edu (Bryan Reed) wrote in message news:<9lgv86$f8o$1...@nntp6.u.washington.edu>...

> In article <9SRe7.3450$2u.3...@www.newsranger.com>,
> Aleksandr Timofeev <a_n_ti...@my-deja.com> wrote:
> >In article <3b7be3bb$0$238$cc9e...@news.dial.pipex.com>, franz heymann says...
> >>
> >>
> >>Aleksandr Timofeev <a_n_ti...@my-deja.com> wrote in message
> >>news:e16a4a22.0108...@posting.google.com...
> >>>
> >>[Snip]
> >>
> >>> I am afraid that we interprete concept " of a free electron "
> >>> by completely different ways.
> >>
> >>Not to mince words, in that case you have got it wrong and I have lost
> >>interest.
> >
> >You can not find the right answer to a so simple problem: What is " a free
> >electron "?
> >
>
>
> I forget how many crackpot points it is for confusing boredom with
> inabiliity to answer . . .
>
> A free electron is one that can be taken to be moving in empty space,

_Empty space_ does not exist in the Nature.

Low energy:

1. You have excluded from consideration background thermal
electromagnetic radiation.
2. You have excluded from consideration various kinds of a substance,
which are present at any cubic centimeter of space in any place
of the Universe.
3. Can you evaluate a radius of an operation (influence) of
a _free electron_, located in rest, on substance, ambient of that one,?


> with (to zeroth order) no externally-applied electric or magnetic fields.

High energy there is other extreme:

4. Very fast driven electron represents a very short electromagnetic
impulse. If such electron is gone in vacuum space inside a metal pipe,
that one is obliged to interact with walls of this pipe (recollect
physical processes in a MICROWAVE waveguides and Maxwell dispacement
currents).

Whether is free such electron?

> So when you're talking about the interaction of a free electron with an
> electromagnetic wave, you're saying the electron is interacting with this
> wave and nothing else.
>

Try to indicate particular examples " of free electrons " from your
point of view.

Have fun,

Aleksandr

It is loading more messages.
0 new messages